Pediatric Dermatology Flashcards

1
Q

1- You are consulted to evaluate a full-term neonate in the newborn nursery and find widespread blotchy erythematous macules, papules, and pustules. Which of the following is most likely to be found upon evaluation of a smear of one of the pustules?

A. Neutrophils
B. Eosinophils
C. Mast cells
D. Bacteria
E. Multinucleated giant cells

A

Correct choice: B. Eosinophils

Explanation: The stem describes the classic presentation of erythema toxicum neonatorum (ETN), which occurs in nearly half of all full term neonates. The pustules are sterile and a smear shows eosinophils. Neutrophils can be found in a smear of transient neonatal pustular melanosis, which is common in darkly pigmented neonates and presents with small pustules or residual hyperpigmented macules with a collarette of scale. Neutrophils and mast cells are not seen in (ETN). The pustules of ETN are sterile, so bacteria are not found. Lastly, multinucleated giant cells are found when performing a Tzanck smear of a vesicle/pustule in patients infected with either HSV or VZV.

How well did you know this?
1
Not at all
2
3
4
5
Perfectly
2
Q

2 -A 15-year-old male with mild intellectual impairment presents with multiple hyperpigmented lesions on the trunk and axillary freckling. Family history was positive for similar lesions in his mother. No ocular findings or cutaneous findings consistent with neurofibromas or plexiform neurofibromas were present. Genetic testing was negative for Neurofibromatosis Type 1. What gene mutation accounts for this patient’s condition?

A. PTPN11
B. RET
C. GNAS
D. SPRED-1
E. Merlin

A

Correct choice: D. SPRED-1

Explanation: This case represents Legius syndrome (Choice 4), which is due to an autosomal dominant mutation in SPRED-1. This syndrome accounts for up to 2% of all patients meeting criteria for NF1. The clinical presentation is similar to that of NF1 with multiple CALMs, axillary or inguinal freckling. Mild intellectual disability, macrocephaly are also commone features. Other NF1-associated features such as Lisch nodules, neurofibromas, NF1-specific bone lesions, optic pathway gliomas, malignant peripheral nerve sheath tumors are absent. PTPN11 (Choice 1) is mutated in LEOPARD syndrome and Noonan syndrome. RET (Choice 2) is mutated in several genodermatoses including MEN 2a and 2b. GNAS (Choice 3) is mutated in McCune Albright syndrome. Merlin (Choice 5) is mutated in NF2.

How well did you know this?
1
Not at all
2
3
4
5
Perfectly
3
Q

3- A newborn has a mottled pinkish blue appearance to the skin that does not change with temperature. Multiple rewarming attempts are futile. What is the most likely associated finding?

A. Significant full body asymmetry
B. Mild ipsilateral limb hypoplasia
C. CNS tumors
D. High-risk of superinfection
E. Severe contralateral foot hyperplasia

A

Correct choice: B. Mild ipsilateral limb hypoplasia

Explanation: The vignettes describes a newborn with cutis mamorata telangiectasia congenita. It is distinct from cutis mamorata which is physiologic. Patients with CMTC may have no associated abnormalities but the most common of the choices here is mild ipsilateral limb hypoplasia. Mild, ipsilateral limb hypoplasia is more likely than contralateral or severe full body asymmetry. CNS tumors and risk of infection would be highly unlikely.

How well did you know this?
1
Not at all
2
3
4
5
Perfectly
4
Q

4- A premature infant who is being weaned off breast milk develops vesicobullous and eczematous skin lesions and diarrhea. Which of the following is NOT another classic precipitant for this condition?

A. Parenteral nutrition
B. Stress (i.e. infection)
C. Diets with mainly cereal grains
D. Liver disease
E. Alcoholism

A

Correct choice: D. Liver disease

Explanation: Zinc deficiency can be seen in premature or term infants being weaned off breast milk, which is usually high in zinc content, as well as in parenteral nutrition use, alcoholism because of poor nutritional intake, malabsorption, IBD, diets high in grains containing phytate which binds zinc, and metabolic stress. Liver disease is not a precipitant for zinc deficiency.

How well did you know this?
1
Not at all
2
3
4
5
Perfectly
5
Q

5- A 6 month-old presents with the follwing lesions around the mouth and groin. Several bullae are also present on the fingers and toes. Which of the following laboratory values is likely to be abnormal?

A. Hematocrit
B. Calcium
C. Platelet count
D. ALT
E. Alkaline phosphatase

A

Correct choice: E. Alkaline phosphatase

Explanation: The most likely diagnosis is acrodermatitis enteropathica. Alkaline phosphatase is a zinc dependant enzyme that is decreased in response to low serum zinc levels. These lab values would not typically be abnormal in acrodermatitis enteropathica.

How well did you know this?
1
Not at all
2
3
4
5
Perfectly
6
Q

6- What is the most likely condition associated with this lesion?

A. McCune-Albright syndrome
B. Neurofibromatosis Type 1
C. Tuberous Sclerosis
D. Neurofibromatosis Type 2
E. This lesion is not associated with any of the above listed conditions

A

Correct choice: E. This lesion is not associated with any of the above listed conditions

Explanation: The image is that of a Becker’s nevus, which is an acquired unilateral hamartomatous lesion found in adolescent males typically on the shoulder, upper chest, or back. It can be distinguished from a cafe-au-lait macule (particularly the “Coast of Maine” cafe-au-lait macule seen in McCune-Albright syndrome) by the hypertrichosis that is not present in a cafe-au-lait. It is not associated with the listed conditions. The other answer choices are not associated with a Becker’s nevus.

How well did you know this?
1
Not at all
2
3
4
5
Perfectly
7
Q

7- A baby boy is presents to your office with his mother who notes he has a ring of hair loss on his scalp. She notes she had a prolonged labor and that this is her first child. Their pediatrician referred him to you for further evaluation and management. On exam, you note annular alopecia. What condition is this associated with?

A. Cephalohematoma
B. Caput succedaneum
C. Sclerema neonatorum
D. Subcutaneous necrosis of the newborn
E. Cutis marmorata

A

Correct choice: B. Caput succedaneum

Explanation: Caput succedaneum presents as localized edema of the scalp due to labor and occurs most frequently in the setting of prolonged labor and primigravidas. On exam, annular alopecia, also known as “halo scalp ring” can be seen due to pressure necrosis. Cephalohematoma is a subperiosteal hematoma that does not cross the midline and can be associated with hyperbilirubinemia.

How well did you know this?
1
Not at all
2
3
4
5
Perfectly
8
Q

8- Which of the following is true regarding neonatal lupus erythematosus?

A. Most cases involve boys
B. Lesions generally resolve spontaneously by 6 months, healing with scarring
C. Photosensitivity is generally not a feature
D. 75% of mothers have symptomatic systemic lupus erythematous at the time of delivery
E. Congenital heart block may be the only manifestation of the disease

A

Correct choice: E. Congenital heart block may be the only manifestation of the disease

Explanation: Neonatal LE presents with annular scaling erythematous macules and plaques on the head and extremities within the first few months of life in babies born to mothers with LE, rheumatic diseases, or other connective tissue disorders. 50% of mothers are asymptomatic at delivery. Lesions resolve spontaneously by 6 months, healing without scarring. Photosensitivity may be prominent. 75% of cases involve girls. 50% have congenital heart block, which is permanent, and may be the only manifestation of the disease. Thrombocytopenia and hepatic disease are as frequent as cardiac disease. The other answer choices listed are incorrect.

How well did you know this?
1
Not at all
2
3
4
5
Perfectly
9
Q

9- What is the most likely diagnosis in this five year old patient?

A. Juvenile xanthogranuloma
B. Benign cephalic histiocytosis
C. Langerhans cell histiocytosis
D. Infantile hemangioma
E. Verruca

A

Correct choice: A. Juvenile xanthogranuloma

Explanation: Juvenile xanthogranuloma is the most likely diagnosis based on the image which depicts a yellow-to-brown papule or plaque on the back of a five year old child. Solitary lesions will regress and are not a cause for alarm. Benign cephalic histiocytosis most commonly presents in infants and manifests with multiple brown yellow papules that spontaneously regress. Langerhans cell histiocytosis presents in several cutaneous forms but is not a cause of a solitary yellow-brown papule or plaque in a 5 year old child. Infantile hemangioma and verruca do not clinically look like the lesion in the photograph.

How well did you know this?
1
Not at all
2
3
4
5
Perfectly
10
Q

11- A 2-month-old female has a red plaque extending from her eye down to her nostril, causing the right nostril to be mildly compressed. The infant is in no acute distress and is playful. What should you discuss with the parents?

A. The lesion will resolve in the next 3 months
B. Growth of the lesion dover the next year is expected and systemic propranolol therapy at weight-based dosing would be optimal but the patient should undergo MRI/MRA of the head, neck, and chest
C. The lesion will grow over 6 months before stopping and topical timolol gel forming solution TID as a solo treatment is all that is necessary
D. The infant has an increased risk of spina bifida
E. The infant will develop coordination and speech problems

A

Correct choice: B. Growth of the lesion dover the next year is expected and systemic propranolol therapy at weight-based dosing would be optimal but the patient should undergo MRI/MRA of the head, neck, and chest

Explanation: The infant has PHACES syndrome with evidence of infantile hemangioma of the eye and nose. Infantile hemangiomas tend to grow during the first year of life prior to involution. In the setting of PHACES syndrome with a large hemangioma over sensitive areas such as the eyes and nose, the patient should initiate systemic propranolol therapy and undergo MRI/MRA of the head, neck, and chest to evaluate for arterial abnormalities. Topical treatment is insufficient for the infantile hemangioma described in this vignette. The lesion will not resolve within 3 months. The infant does not have an increased risk of spina bifida and will not necessarily develop any coordination or speech problems.

How well did you know this?
1
Not at all
2
3
4
5
Perfectly
11
Q

12- Criteria for PHACE syndrome includes presence of a facial hemangioma >5cm + 1 major or 2 minor criteria. Which of the following is a major criterion of PHACE syndrome that could be used in combination with the large hemangioma to clinch the diagnosis?
A. Intracranial hemangioma
B. Ventricular septal defect
C. Posterior segment ocular anomalies
D. Anterior segment ocular anomalies
E. Hypopituitarism

A

Correct choice: C. Posterior segment ocular anomalies

Explanation: Major diagnostic criteria for PHACE syndrome include anomalies of major cerebral arteriaes, posterior fossa anomalies, aortic arch anomalies/aberrant subclavian artery origin, posterior segment ocular anomalies, and sternal abnormalities (clefting, pits, papules)/ supraumbilical raphe. Intracranial hemangiomas, ventricular septal defects, anterior segment ocular anomalies and hypopituitarism are all minor criteria.

How well did you know this?
1
Not at all
2
3
4
5
Perfectly
12
Q

13- This healthy 2-year-old female presents with a 2-day history of the pruritic rash seen here. She had mild edema of the dorsal hands and feet and positive dermatographism on exam. All lesions were transient, lasting less than 24 hours in the same location. She was otherwise well and her parents denied any fever or systemic symptoms. What is the best diagnosis?

A. Serum sickness-like reaction
B. Erythema multiforme
C. Staphylococcal scalded skin syndrome
D. Urticaria multiforme
E. Morbilliform drug eruption

A

Correct choice: D. Urticaria multiforme

Explanation: The physical exam and findings suggest urticaria multiforme. The patient expresses pruritus, acral edema, positive dermatographism, and transient lesions and importnatly she is otherwise well. Serum sickness-like reaction the patient is more ill, and has had recently been exposed to antibiotics. The patient has fever, joint pain, etc.
Erythema multiforme is less pruritis, lesions are not transient, and they are often less edematous than those seen here. Staphylococcal scalded skin syndrome is an exfoliative infectious disorder in which there is superficial desquamation. A morbilliform drug eruption will have a medication history and it will lack the annular, edematous plaques seen here.

How well did you know this?
1
Not at all
2
3
4
5
Perfectly
13
Q

14- Which of the following encompasses the most common extracutaneous manifestation of PHACE syndrome?

A. Structural brain anomalies
B. Cerebrovascular anomalies
C. Cardiac anomalies
D. Ocular anomalies
E. Sternal clefting

A

Correct choice: B. Cerebrovascular anomalies

Explanation: Haggstrom et al. observed that the most common extracutaneous findings in PHACE syndrome were abnormalities in the cerebrovasculature (91%) followed by cardiac anomalies (67%). Pediatrics; 2010

How well did you know this?
1
Not at all
2
3
4
5
Perfectly
14
Q

-A 4-year-old boy presents to your clinic having recently returned from visiting family in the Philippines. His parents report that he has fever, cough, and nasal congestion. On exam, you notice conjunctival injection and the lesions pictured below. He does not have a rash. What is the most likely diagnosis?

A. Kawasaki disease
B. Rubeola
C. Hand-Foot-Mouth disease
D. Herpangina
E. Rubella

A

Correct choice: B. Rubeola

Explanation: This boy is presenting with typical findings of rubeola (measles or first disease). It is important to note that in measles, an enanthem (Koplik spots are pictured in the image) precedes the development of the classic exanthem: erythematous macules and papules beginning over the forehead, hairline, and behind the ears with subsequent downward progression. The remaining listed answer choices do not display Koplik spots.

How well did you know this?
1
Not at all
2
3
4
5
Perfectly
15
Q

16- The infant shown in this image is at risk for which of the following complications?

A. Cleft palate
B. Diaphragmatic hernia
C. Hypospadias
D. Imperforate anus
E. Coarctation of the aorta

A

Correct choice: E. Coarctation of the aorta

Explanation: This patient has a segmental infantile hemangioma on the face and should be evaluated for PHACES syndrome: posterior fossa malformations, hemangiomas, arterial anomalies, cardiac defects and coarctation of the aorta (Choice 5), eye anomalies, sternal defects and supraumbilical raphe. Work-up includes MRI/MRA of the head and neck, referral to a cardiologist, and referral to a pediatric ophthalmologist. Cleft palate (Choice 1), diaphragmatic hernia (Choice 2), hypospadias (Choice 3), imperforate anus (Choice 4) are not associated with PHACES syndrome.

How well did you know this?
1
Not at all
2
3
4
5
Perfectly
16
Q

17- A 20-year-old male presents with jaundice, hepatomegaly, blue lunulae, and the ocular finding as shown. What is the genetic mutation in this condition?

A. Phenylalanine hydroxylase (PAH) gene
B. α-galactosidase A (GLA) gene
C. ATP7B gene
D. Glucocerebrosidase (GBA) gene
E. Cystathionine β-synthase (CBS) gene

A

Correct choice: C. ATP7B gene

Explanation: The correct answer is the ATP7B gene (Choice 3), which is mutated in Wilson’s disease. In this condition, there is a defect in copper metabolism leading to deposition in the liver and subsequent liver failure. Cutaneous findings include Kayser-Fleischer rings (due to copper deposition in Descemet’s membrane), blue lunulae, and pretibial hyperpigmentation. Phenylalanine hydroxylase (PAH) gene (Choice 1) is mutated in phenylketonuria. α-galactosidase A (GLA) gene (Choice 2) is mutated in Fabry disease. Glucocerebrosidase (GBA) gene (Choice 4) is mutated in Gaucher disease. Cystathionine β-synthase (CBS) gene (Choice 5) is mutated in homocystinuria.

How well did you know this?
1
Not at all
2
3
4
5
Perfectly
17
Q

18- Which of the following is more common in the juvenile variant of the skin disease depicted here?

A. Calcinosis cutis
B. Poikiloderma
C. Anti-Jo antibodies
D. Gottron’s papules
E. Associated malignancy

A

Correct choice: A. Calcinosis cutis

Explanation: Calcinosis cutis is more common in juvenile dermatomyosits (compared to the adult population). The other options are not more common in pediatric dermatomyositis.

How well did you know this?
1
Not at all
2
3
4
5
Perfectly
18
Q

19- A 10-year-old girl presents with multiple ill-defined hypo pigmented macules and patches, ranging from 0.5 to 2 cm in diameter with fine scaling. Her lesions are located on both cheeks. Which of the following conditions describes this clinical picture, which is thought to result from a

low-grade eczematous dermatitis that disrupts the transfer of melanosomes from melanocytes to keratinocytes?

A. Pityriasis lichenoides chronica
B. Pityriasis alba
C. Vitiligo
D. Tinea versicolor
E. Hypopigmented mycosis fungoides

A

Correct choice: B. Pityriasis alba Explanation:
Pityriasis alba frequently affects children and adolescents with AD. It is characterized by multiple ill-defined hypopigmented macules and patches, usually 0.5 to 2 cm in diameter, with fine scaling; the lesions are typically located on the face, especially the cheeks, but occasionally appear on the shoulders and arms. Pityriasis alba is most obvious in individuals with darkly pigmented skin and/or following sun exposure. It is thought to result from a low-grade eczematous dermatitis that disrupts the transfer of melanosomes from melanocytes to keratinocytes. Pityriasis lichenoides chronica: Postinflammatory hypopigmentation secondary to other dermatoses such as seborrheic dermatitis or pityriasis lichenoides chronica can occur but the distribution is more widespread in pityriasis lichenoides chronica - the lesions would not just appear on the cheeks. Vitiligo: Vitiligo is sharply demarcated and depigmented rather than hypopigmented. Tinea versicolor: Pityriasis (tinea) versicolor is typically more sharply demarcated, with small lesions that may coalesce centrally in involved areas. Hypopigmented mycosis fungoides: Hypopigmented mycosis fungoides may occasionally represent a diagnostic consideration if there is extrafacial involvement.

How well did you know this?
1
Not at all
2
3
4
5
Perfectly
19
Q

20- A 3-year-old boy presented with a generalized eruption of numerous tan to brown macules and papules on the trunk and limbs. He felt more itchy when he was hot or after scratching. The lesions noticeably produced an urticarial wheal after rubbing. The patient was otherwise healthy. Which of the following should be avoided in this condition?

A. Hydroxyzine
B. Prednisone
C. Morphine
D. Mupirocin
E. Cyproheptadine

A

Correct choice: C. Morphine

Explanation: This case describes urticaria pigmentosa, the most common childhood form of mastocytosis. Patients present with multiple light brown to red-brown macules and papules. Pruritus and flushing may be seen and Darier’s sign is positive (urticarial wheal formation after friction or rubbing). It is important for patients with urticaria pigmentosa to avoid mast cell degranulators e.g. alcohol, anticholinergics, NSAIDs, aspirin, narcotics e.g. morphine (Choice 3) and codeine, polymyxin, and systemic anesthetics. Hydroxyzine (Choice 1), prednisone (Choice 2), mupirocin (Choice 4), and cyproheptadine (Choice 5) are not mast cell degranulators.

How well did you know this?
1
Not at all
2
3
4
5
Perfectly
20
Q

21- What imaging study is best to confirm PHACE syndrome?

A. Cerebral ultrasound
B. MRI/MRA
C. CT Scan
D. Fundoscopy
E. Serial xray

A

Correct choice: B. MRI/MRA

Explanation: The best imaging test looking for associated cerebral anomalies in patients with PHACE syndrome is MRI/MRA.

How well did you know this?
1
Not at all
2
3
4
5
Perfectly
21
Q

22- Subcutaneous fat necrosis of the newborn has been associated with:
A. Hypocalcemia
B. Hypercalcemia

C. Hypokalemia
D. Hyperkalemia
E. Hyponatremia

A

Correct choice: B. Hypercalcemia

Explanation: Hypercalcemia has been noted in some cases of subcutaneous fat necrosis of the newborn. Because the onset of hypercalcemia may be delated, serum calcium levels should be monitored for several months after the diagnosis. The other listed electrolyte abnormalities are not commonly reported in the setting of subcutaneous fat necrosis of the newborn.

How well did you know this?
1
Not at all
2
3
4
5
Perfectly
22
Q

23- A 31-year-old male with history of recurrent febrile episodes presents with the following skin findings. The gene that is mutated in this condition encodes for what type of protein?

A. Tyrosine kinase receptor
B. Protein tyrosine protein phosphatase
C. Telomerase complex
D. DNA helicase
E. Nuclear envelope protein

A

Correct choice: C. Telomerase complex

Explanation: The correct answer is the telomerase complex (Choice 3), as this case represents dyskeratosis congenita. Dyskeratosis congenita occurs due to mutations in TERT (AD/AR), TERC (AD), DKC1 (XLR), or TINF2 (AD) genes which encode for proteins involved in telomere maintenance. Affected patients manifest reduced telomerase activity and abnormally shortened telomeres leading to chromosomal instability and cellular replication dysfunction. Clinical features include bone marrow failure and the triad of abnormal reticulated hyperpigmentation on the face/

neck/upper trunk, oral leukoplakia (premalignant), and onychodystrophy (pterygium, anonychia, longitudinal ridging/splitting). Multiple endocrine neoplasia (MEN) type 2 is due to a mutation in the RET proto-oncogene, which encodes for a tyrosine kinase receptor (Choice 1). Noonan syndrome is due to a mutation in PTPN11, which encodes a protein tyrosine protein phosphatase (Choice 2). Rothmund-Thompson Syndrome is due to a mutation in RecQL4, which encodes a DNA helicase (Choice 4). Progeria is due to a mutation in Lamin A, which encodes a nuclear envelope protein (Choice 5).

How well did you know this?
1
Not at all
2
3
4
5
Perfectly
23
Q

24- You are consulted on a 1 week-old baby girl who has developed a new rash. Birth history was uncomplicated and she was born at full-term. The team notes the neonate began to develop several indurated plaques on her back, buttocks, and thighs. What laboratory abnormality is associated with this condition?

A. Neutropenia
B. Thrombocytopenia
C. Hypocalcemia
D. Hypercalcemia
E. Hypernatremia

A

Correct choice: D. Hypercalcemia

Explanation: This neonate has subcutaneous fat necrosis of the newborn, which presents with localized subcutaneous nodules on the buttocks, thighs, arms, face and shoulders. It typically occurs within the first weeks of life in healthy term infants. Calcification is common and it is associated with a profound hypercalcemia. Calcium levels should be followed until one month after all lesions have cleared. The remaining lab abnormalities are not commonly associated with subcutaneous fat necrosis of the newborn.

How well did you know this?
1
Not at all
2
3
4
5
Perfectly
24
Q

25- Which of the following disorders is more likely to occur in children with chronic fecal incontinence?

A. Langerhans cell histiocytosis
B. Herpes simplex infection
C. Perianal streptococcal disease

D. Granuloma gluteale infantum
E. Seborrheic dermatitis

A

Correct choice: D. Granuloma gluteale infantum

Explanation: Granuloma gluteale infantum, which is considered to be a form of chronic irritant diaper dermatitis, is seen more commonly in the setting of chronic fecal incontinence. Langerhans cell histiocytosis, herpes simplex infection, perianal streptococcal disease, and seborrheic dermatitis are not seen more often in the setting of chronic fecal incontinence.

How well did you know this?
1
Not at all
2
3
4
5
Perfectly
25
Q

26- Sturge-Weber syndrome is a sporadic disorder characterized by a facial port-wine stain associated with ocular and neurologic abnormalities. It can occur in the first year of life and is more common in patients with bilateral forehead port-wine stains. Which of the following is the most common neurologic abnormality in patients with Sturge-Weber syndrome?

A. Seizures
B. Motor dysfunction
C. Glaucoma
D. Intracranial “tram-track” calcification
E. Mental retardation

A

Correct choice: A. Seizures

Explanation: Of the answer choices listed, the most common neurologic abnormality that patients with Sturge-Weber syndrome develop is seizures. Motor dysfunction, intracranial “tram-track” calcification, and mental retardation may occur in patients with Sturge-Weber syndrome, however seizures are more common. Glaucoma is an extremely common OCULAR abnormality

How well did you know this?
1
Not at all
2
3
4
5
Perfectly
26
Q

27- This syndrome is characterized by a nevus sebaceous associated with ocular malformations, mental retardation, seizures, and skeletal and cardiovascular abnormalities:
A. Schimmelpenning syndrome
B. Nevus comedonicus syndrome
C. Nevus sebaceous of Jadassohn

D. Ichthyosis hystrix
E. Schnitzler syndrome

A

Correct choice: A. Schimmelpenning syndrome

Explanation: Schimmelpenning syndrome (aka organoid nevus syndrome) involves a mostly linear nevus sebaceous associated with ocular malformations, mental retardation, seizures, and skeletal and cardiovascular abnormalities. Nevus sebaceous is a congenital hairless, yellow to orange plaque on the scalp. Nevus comedonicus syndrome is characterized by a nevus comedonicus plus ipsilateral cataract and skeletal/CNS abnormalities. Nevus sebaceous of Jadassohn refers simply to a solitary nevus sebaceous. Ichthyosis hystrix (aka generalized epidermal nevus) displays extensive bilateral systematized lesions. Lastly, Schnitzler syndrome is typified by episodes of urticaria plus fever, lymphadenopathy, hepatosplenomegaly, bone pain, sensorimotor neuropathy, and a monoclonal IgM gammopathy.

How well did you know this?
1
Not at all
2
3
4
5
Perfectly
27
Q

28- Which of the following lesions demonstrates a pseudo-Darier’s sign?

A. Mastocytoma
B. Spitz nevus
C. Smooth muscle hamartoma
D. Pilomatricoma
E. Bullous pemphigoid

A

Correct choice: C. Smooth muscle hamartoma

Explanation: Smooth muscle hamartomas are benign tumors which arise from smooth muscle of the dermis. Pseudo-Darier’s sign may be elicited due to transient piloerection after rubbing. Histologically, red-orange bundles and fascicles are present with blunt-ended nuclei. Pseudo- Darier’s sign is not seen with the other listed conditions.

How well did you know this?
1
Not at all
2
3
4
5
Perfectly
28
Q
  1. A- 4-month-old African-American male presents for the rash shown. The rash has been present for the past two months. The mother describes that the baby tends to be bothered by this rash and can be seen trying to rub his feet together. Which of the following tests should be performed next?

A. KOH
B. Mineral oil prep
C. Skin biopsy
D. Bacterial culture
E. Fungal culture

A

Correct choice: B. Mineral oil prep

Explanation: This clinical image and scenario most likely represents Infantile acropustulosis (IA), which is a benign vesiculopustular condition with an often chronic course in infants. The etiology is unknown. Some studies suggest that African-American male infants are more commonly affected. IA is characterized by recurrent crops of intensely pruritic vesiculopustules on the palms and soles. The diagnosis is based on clinical features and given the similar appearance, scabies should be ruled-out with a mineral oil prep. The other tests are generally not necessary in this clinical scenario. If a skin biopsy is performed, it would demonstrate a subcorneal pustule with neutrophils and eosinophils.

How well did you know this?
1
Not at all
2
3
4
5
Perfectly
29
Q

30- What is the diagnosis?

A. Eosinophilic pustular folliculitis
B. Verrucae vulgares
C. Molluscum contagiosum

D. Herpes simplex
E. Cryptococcus

A

Correct choice: C. Molluscum contagiosum

Explanation: Molluscum contagiosum is a viral infection characterized by skin-colored, dome- shaped papules with central umbilication. The diagnosis can be made clinically. A benign and self- limited skin condition, treatment options include destructive modalities or observation. Inflammation of individual lesions may indicate spontaneous regression. Eosinophilic pustular folliculitis is a rare condition of infancy characterized by sterile pustules most commonly located on the face and scalp. The lesions are pruritic and tend to develop heme crusts. This form is not associated with HIV. Verrucae vulgares are hyperkeratotic, exophytic papules also known as common warts. They are the result of cutaneous infection with the human papillomavirus. Central umbilication is not a characteristic feature. Herpes simplex is caused by herpes simplex virus 1 and
2. The characteristic morphology is grouped vesicles on an erythematous base. In the immunocompromised patient, the differential diagnosis of umbilicated papules includes Cryptococcus. Systemic symptoms, as well as rapid spreading, ulceration and bleeding, may be important clinical clues.

How well did you know this?
1
Not at all
2
3
4
5
Perfectly
30
Q

31- This syndrome has aplasia cutis congenita, cutis marmorata telangiectatica congenita, limb defects and cardiac abnormalities:

A. Adams-Oliver Syndrome
B. Gianotti-Crosti Syndrome
C. Bart Syndrome
D. Seitles Syndrome
E. McCune-Albright syndrome

A

Correct choice: A. Adams-Oliver Syndrome

Explanation: Adams-Oliver syndrome is characterized by aplasia cutis congenita on the scalp (with skull ossification defect), cutis marmorata telangiectatica congenita, limb defects and cardiac abnormalities. Aplasia cutis congenital (ACC) is characterized by an absence of skin and subcutaneous tissue. Gianotti-Crosti syndrome manifests as an abrupt onset of skin-colored to pink- red edematous papules on the cheeks, buttocks, and extremities. It is typically due to a viral infection (EBV, HBV). Bart syndrome is characterized by ACC of the lower extremities plus epidermolysis bullosa (typically dominant dystrophic EB). Seitles syndrome displays bilateral temporal ACC, abnormal eyelashes, leonine facies, and upward-slanting eyebrows. McCune- Albright syndrome (aka polyostotic fibrodysplasia) results from a sporadic mutation in GNAS, and is characterized by precocious puberty, endocrine hyperfunction, large cafe-au-lait pigmentation (“coast of Maine”), and fibrous dysplasia of bones (which may lead to pathologic fractures).

How well did you know this?
1
Not at all
2
3
4
5
Perfectly
31
Q

32- A child presents with a 1 cm yellow-red nodule on the face. Pathology shows Touton giant cells. What is the most frequent site of extracutaneous involvement in this disease?

A. Eye
B. Lung
C. Bone
D. CNS
E. Visceral

A

Correct choice: A. Eye

Explanation: The eye is the most frequent site of extracutaneous juvenile xanthogranuloma. Ocular involvement is typically unilateral. The second most common site of extracutaneous disease is the lungs.

How well did you know this?
1
Not at all
2
3
4
5
Perfectly
32
Q

33- A 75-year-old man presents with the following lesion on the right anterior neck. He states it has been present since birth or early infancy. What is the most likely diagnosis?

A. Verruca vulgaris

B. Spitz nevus
C. Aplasia cutis congenta
D. Lipoma
E. Epidermal nevus

A

Correct choice: E. Epidermal nevus

Explanation: This is a example of a verruous somewhat linear plaque present since birth or early infancy, which is most consistent with an epidermal nevus. A verruca would not be present for the patient’s entire lifetime. The other options would not present as a verrucous plaque.

How well did you know this?
1
Not at all
2
3
4
5
Perfectly
33
Q

34- The most common type of pityriasis rubra pilaris in childhood is type:

A. I
B. II
C. III
D. IV
E. V

A

Correct choice: D. IV

Explanation: Type IV, or circumscribed juvenile, accounts for 25% of total PRP cases, and is the most common type of PRP in childhood. The most common type of PRP is Type I (classic adult), which accounts for 55% of cases. Types II (atypical adult), III (classic juvenile), and V (atypical juvenile) each account for 10% or less of cases.

How well did you know this?
1
Not at all
2
3
4
5
Perfectly
34
Q

35- A mutation in which of the following has been associated with both Sturge-Weber Syndrome and port-wine stains?

A. GNAQ
B. GNAS
C. PIK3CA
D. PTEN

E. RASA1

A

Correct choice: A. GNAQ

Explanation: The Sturge–Weber syndrome and port-wine stains are caused by a somatic activating mutation in GNAQ. GNAS mutations are associated with McCune-Albright syndrome and Albright hereditary osteodystrophy. PIK3CA mutations are associated with Klippel-Trenaunay syndrome, MCAP and CLOVES syndromes, as well as fibroadipose hyperplasia. PTEN mutations are associated with Cowden syndrome, Bannayan-Riley-Ruvalcaba syndrome, Proteus-like syndrome, and Lhermitte-Duclos disease. RASA1 mutations are associated with Parkes-Weber syndrome.

How well did you know this?
1
Not at all
2
3
4
5
Perfectly
35
Q

36- Which of the following virus families is responsible for the lesions shown on this 3-year-old male’s chest?

A. Herpes simplex virus
B. Human papillomavirus
C. Paramyxovirus
D. Poxvirus
E. Togavirus

A

Correct choice: D. Poxvirus

Explanation: The image shows molluscum contagiosum, which is a member of the Poxvirus family. Molluscum does not belong to the other listed families of viruses.

How well did you know this?
1
Not at all
2
3
4
5
Perfectly
36
Q

37- What is the diagnosis?

A. Linear psoriasis
B. Lichen striatus
C. Verrucae
D. Herpes zoster
E. Epidermoid nevus

A

Correct choice: B. Lichen striatus

Explanation: Lichen striatus is a benign and self limited inflammatory condition, most commonly affecting pediatric patients. The flat topped papules form Blashko’s lines. The dermatitis is usually asymptomatic and treatment is not necessary. When an extremity is involved, there may be associated nail dystrophy. Psoriasis is a papulosquamous dermatosis; there is a linear variant. The primary lesion of psoriasis is a discrete erythematous papule or plaque with micaceous scale. In the inverse form, scale is less apparent. Verrucae are hyperkeratotic papules cause by the human papillomavirus. Due to autoinoculation, at times verrucae can be arranged linearly, particularly with flat warts. Thrombosed capillaries may be seen as black dots, aiding in the daignosis. The primary lesions of herpes zoster are grouped vesicles on an erythematous base in a dermatomal arrangement. An epidermal nevus is a benign growth composed of epidermal structures. It is usually present at birth or recognized in early childhood. It forms Blashko’s lines like lichen straitus and can be distinguisehd by its hyperpigmented and verrucous appearance.

How well did you know this?
1
Not at all
2
3
4
5
Perfectly
37
Q

38- A 4-year-old male presents with lesions on the hands, and feet with small papulovesicles with surrounding mild erythema. Mom reports he has not been eating well, and you see a few small gray- white vesicles and shallow erosions on the hard and soft palate as well. Mom is concerned about the course and complications. What is true regarding counseling of sequelae of this entity?

A. Brittle hair often results 2-3 weeks after the eruption subsides
B. There is a very high association with recurrent EBV and viral titers must be followed
C. Risk of testicular involvement is high and close monitoring is recommended

D. Severe birth defects have been seen when women are exposed in early pregnancy so contact and respiratory isolation is needed
E. Nail shedding, onychomadesis, can occur but results in normal nail plate growth

A

Correct choice: E. Nail shedding, onychomadesis, can occur but results in normal nail plate growth

Explanation: This vignette describes hand-foot-mouth syndrome. A common finding after the viral illness is to have shedding of the fingernails, onychomadesis. Afterwards the nails grow back normally and reassurance is important for patients and parents. The other options are true of other viral illnesses but not of hand-foot-mouth particularly.

How well did you know this?
1
Not at all
2
3
4
5
Perfectly
38
Q

39- A 3 month old presents with a diffuse vesiculobullous rash with copper colored macules on the palms and soles. You ascertain from the history that his mother had a nonpainful erosion on her labia during pregnancy which spontaneously resolved. What other symptoms would you expect this infant to have?

A. Pseudoparalysis of Parrot
B. Higoumenakis sign
C. Clutton joints
D. Mulberry molars
E. Saddle nose deformity

A

Correct choice: A. Pseudoparalysis of Parrot

Explanation: Pseudoparalysis of Parrot is a sign of early congenital syphilis. Early congenital syphilis occurs before 2 years of age, whereas late congenital syphilis generally occurs after 2 years of age. Higoumenakis sign, clutton joints, mulberry molars, and saddle nose deformity are all signs of late congenital syphilis.

How well did you know this?
1
Not at all
2
3
4
5
Perfectly
39
Q

40- A concerned couple presents to your pediatric dermatology clinic with their newborn infant who has a non-inflamed superficial ulceration on the dorsal hand, present since birth. The patient is otherwise healthy. Based on the clinical description and age of the patient, you tell the parents:

A. This lesion may be caused by vigorous sucking of the affected region in utero
B. Common locations include the head and neck
C. This condition is usually associated with many genetic abnormalities
D. The ulceration will resolve in months to years
E. Therapy involves systemic corticosteroids

A

Correct choice: A. This lesion may be caused by vigorous sucking of the affected region in utero

Explanation: A sucking blister is caused by vigorous sucking of the affected region in utero. Common locations include the radial forearm, wrists, hands, and fingers. It may present with an intact bulla, erosion, callus or superficial ulceration on a non-inflamed base. It is usually an isolated finding without associated abnormalities. It resolves over days to weeks and therapy is usually supportive. 2 – Sucking blisters are commonly located on the radial forearm, wrist, hands and fingers. 3 – Sucking blisters are usually an isolated finding without associated abnormalities. 4 – Sucking blisters resolve over days to weeks and therapy is supportive. 5 – Sucking blisters resolve over days to weeks and therapy is supportive.

How well did you know this?
1
Not at all
2
3
4
5
Perfectly
40
Q

41- How are these lesions typically distributed?

A. Along the milk line
B. On the scalp
C. Along the lines of blaschko
D. On the trunk
E. On the extremities

A

Correct choice: C. Along the lines of blaschko

Explanation: This is an epidermal nevus. Epidermal nevi are typically distributed along the lines of blaschko and can be located anywhere on the body

How well did you know this?
1
Not at all
2
3
4
5
Perfectly
41
Q

42- Which of the following is not a major criterion for Kawasaki’s disease:
A. Bilateral conjunctival injection
B. Palmoplantar erythema > desquamation
C. Cardiac aneurysm
D. Strawberry tongue/ red lips
E. Cervical adenopathy

A

Correct choice: C. Cardiac aneurysm

Explanation: Cardiac aneurysm is a serious complication of . However, as the cardiovascular manifestations generally present 1 –5 months after presentation, they are not criteria for diagnosis. The other listed choices are major criterion for Kawasaki’s disease.

How well did you know this?
1
Not at all
2
3
4
5
Perfectly
42
Q

43- An infant is referred by their pediatrician for further evaluation of seborrheic dermatitis refractory to standard therapy. What is the most appropriate intervention?

A. Prescribe clobetasol solution
B. Obtain bacterial culture
C. Obtain fungal culture
D. Obtain complete blood count
E. Obtain skin biopsy

A

Correct choice: E. Obtain skin biopsy
Explanation: In infants, Langerhans cell histiocytosis (LCH) can mimic severe seborrheic dermatitis. A skin biopsy should be performed to make the diagnosis.
This question tests the examinee’s ability to recognize LCH presenting with coalescing erythematous papules with secondary crusting and erosion on an infant’s scalp, and know the correct next step in management. A skin biopsy (choice 5) is required to make the diagnosis of

LCH. Prescribing a strong topical steroid solution (choice 1) is generally inappropriate in infants and will not assist in making the diagnosis imminently needed in this scenario. Although secondary impetiginization may be present, obtaining a bacterial culture (choice 2) is not the best answer because it will delay the diagnosis of LCH, a potentially life-threatening condition. The favus form of dermatophyte hair infection can produce large yellow crusts on the scalp, but would not be expected to demonstrate discrete erythematous papules; therefore, fungal culture (choice 3) is not the best answer. A complete blood count (choice 4) is obtained as part of the workup for LCH, but is of lower priority than skin biopsy to make the initial diagnosis.

How well did you know this?
1
Not at all
2
3
4
5
Perfectly
43
Q

44- A healthy neonate presents with discrete, scattered, 1-3mm yellowish papules on the face, trunk, arms, and legs. An erythematous flare surrounds the papules. Palms and soles are spared. Papules started on the second day of life, and some initial papules have already resolved. Which of the following would you recommend?

A. Acyclovir
B. Avoidance of heat
3. Fluconazole
D. Penicillin
E. Reassurance

A

Correct choice: E. Reassurance

Explanation: This case describes erythema toxicum neonatorum, a benign, self-limited neonatal eruption that spares the self-limited neonatal eruption that spares the palms and soles that affects 7-45% of newborns. The condition typically presents on the second or third day of life and resolves by approximately 1 week. No treatment is indicated. Choice 1 is incorrect as acyclovir would be the treatment of neonatal herpes simplex virus (HSV) infection. Choice 2 is incorrect as avoidance of heat would be the treatment for miliaria. Choice 3 is incorrect as fluconazole would be indicated for neonatal candidiasis. Choice 4 is incorrect as penicillin would be the treatment for neonatal syphilis.

How well did you know this?
1
Not at all
2
3
4
5
Perfectly
44
Q

45- A 7-year-old male returns from a camping trip to Tennessee and develops fever, headache, malaise and myalgias. He then develops a macular rash on his ankles and wrists. Given the high concern for Rocky Mountain Spotted Fever, what is the appropriate treatment?

A. Amoxicillin
B. Azithromycin
C. Chloramphenicol
D. Doxycycline
E. Trimethoprim-sulfamethoxazole

A

Correct choice: D. Doxycycline

Explanation: The preferred treatment for all patients with Rocky Mountain Spotted Fever is doxycycline. Although tetracyclines can cause dental staining when administered to children younger than eight years old, the risk of staining is minimal if a short course is prescribed. In a study of 53 children who received doxycycline for RMSF, none developed dental staining. The other medications are not first-line for RMSF.

How well did you know this?
1
Not at all
2
3
4
5
Perfectly
45
Q

46- A 7-year-old girl presents to your clinic with a multi-day history of fever, sore throat, and dusky vesicles, pustules, and erosions on the hands, feet, and buttocks. Oral examination reveals multiple erosions with surrounding erythema. Which of the following is the most likely causative organism?

A. Enterovirus 71
B. Parvovirus B19
C. Herpes simplex virus 1
D. Coxsackievirus A16
E. Epstein-Barr virus

A

Correct choice: D. Coxsackievirus A16

Explanation: The question stem describes the classic presentation of Hand-Foot-Mouth (and Butt) disease, which is a self-limited condition that commonly occurs in children <10 years (+/- adults). Coxsackievirus A16 is the most likely causative organism. It is important to note that Coxsackievirus A6 may lead to development of atypical HFM disease, which is more widespread and severe. Enterovirus 71 may cause HFM disease, but it is less likely than Coxsackievirus A16. Parvovirus B19 causes erythema infectiosum (Fifth disease), hydrops fetalis, and papular purpuric gloves and socks syndrome. Herpes simplex virus 1 most commonly causes herpes gingivostomatitis, and may also cause genital herpes (HSV 2 is more likely though). Epstein-Barr

virus is associated with infectious mononucleosis and Gianotti-Crosti syndrome, among other conditions.

How well did you know this?
1
Not at all
2
3
4
5
Perfectly
46
Q

47- The treatment for acrodermatitis enteropathica is:

A. Zinc supplementation
B. Iron supplementation
C. Vitamin B1 supplementation
D. Vitamin B12 supplementation
E. Phlebotomy

A

Correct choice: A. Zinc supplementation

Explanation: Acrodermatitis enteropathica is due to a defect in zinc absorption and will respond to zinc supplementation. Findings include periorificial, scalp, and acral dermatitis, scaling, vesicles/ bullae, erosions, alopecia, diarrhea and stomatitis. Iron, Vitamin B1/12 supplementation will not result in improvement in this condition, neither will phlebotomy.

How well did you know this?
1
Not at all
2
3
4
5
Perfectly
47
Q

48- A 10-year-old male presents with recurrent crops of hypopigmented macules and crusted papules on the trunk and extremities. What is the hypothesized pathogenesis of this disorder?

A. CD4+ cell infiltrate
B. HHV-6/HHV-7
C. Malassezia
D. Deficiency in vitamin A
E. Preceding streptococcal pharyngitis

A

Correct choice: A. CD4+ cell infiltrate

Explanation: The image shows pityriasis lichenoides chronica (PLC), a rare, chronic cutaneous disorder most commonly seen in children and young adults with a slight male predominance. PLC presents with a widespread eruption comprised of scaly, red-brown papules and plaques that resolve with hypopigmentation. Both pityriasis lichenoides at varioliformis acute (PLEVA) and PLC contain lesional T cell infiltrates, with a general predominance of CD8+ T cells in PLEVA and CD4+ cells in PLC (Choice 1). Choice 2 is incorrect as this describes pityriasis rosea, which has been associated with reactivation of HHV-6/HHV-7. Choice 3 is incorrect as this describes tinea versicolor, which is caused by growth of dimorphic yeast of the Malassezia genus. Choice 4 is incorrect as this describes phrynoderma, which is due to deficiency in vitamin A metabolism. Choice 5 is incorrect as this describes guttate psoriasis, which has been associated with preceding streptococcal pharyngitis.

How well did you know this?
1
Not at all
2
3
4
5
Perfectly
48
Q

49- Which of the following is true regarding this lesion?

A. It usually onsets in adulthood
B. It most commonly presents in those of African descent
C. It is typically bilateral
D. The sclera is the most common extracutaneous site
E. It is more common in men

A

Correct choice: D. The sclera is the most common extracutaneous site

Explanation: The image depicts a Nevus of Ota, which presents as unilateral, grey-blue macules typically involving the V1 and V2 distributions of the trigeminal nerve. The most common extracutaneous site is the sclera, followed by the typanum. Nevus of Ota onsets either near birth or during puberty, occurs most commonly in women of Asian descent, and is most frequently unilateral.

How well did you know this?
1
Not at all
2
3
4
5
Perfectly
49
Q

50- The dermatologist on call is requested to visit the NICU to evaluate an infant born 12 hours earlier with a disseminated rash. Two punch biopsies are performed and a c-KIT positive infiltrate is noted in the dermis of each sample. What is the most likely diagnosis?

A. T cell lymphoma
B. Mastocytosis
C. Syphilis, congenital
D. B cell lymphoma
E. Neuroblastoma

A

Correct choice: B. Mastocytosis

Explanation: The patient presents with likely systemic mastocytosis, but definitively with diffuse cutaneous mastocytosis based on the appearance of the skin lesions and the c-KIT positive staining pattern. T cell and B cell lymphoma are not correct as the staining pattern and infiltrate would appear differently in each of those conditions. Congenital syphilis would not stain positive for c- KIT, though syphilis should be on the differential diagnosis clinically. Neuroendocrine IHC markers, such as CD56, chromogranin A, and synaptophysin would be present in the case of neuroblastoma.

How well did you know this?
1
Not at all
2
3
4
5
Perfectly
50
Q

51- What is the most likely diagnosis?

A. Incontinentia pigmenti
B. Bullous pemphigoid of infancy
C. Herpes zoster
D. Disseminated herpes simplex
E. Epidermolysis bullosa simplex

A

Correct choice: A. Incontinentia pigmenti
Explanation: The vesicular lesions following the lines of Blaschko are typical for incontinentia pigmenti. The other listed conditions are less likely to be represented by this clinical photo.

How well did you know this?
1
Not at all
2
3
4
5
Perfectly
51
Q

52- A teenager presents to your surgery clinic for removal of a cyst in the suprasternal notch which often drains fluid. Upon examining the excision on histology you notice thick ciliated epithelium. The most likely developmental defect in this case is:

A. Branchial cleft cyst
B. Branchial cleft sinus
C. Thyroglossal cyst
D. Thyroglossal sinus
E. Bronchogenic cyst

A

Correct choice: E. Bronchogenic cyst
Explanation: Bronchogenic cysts are most commonly found in the suprasternal notch and often drain fluid. The lining of the cyst is typically ciliated and may be columnar or cuboidal. Squamous metaplasia is common. In the wall, it is very common to find smooth muscle and respiratory-type mucous glands. Cartilage is another common finding in the wall.

How well did you know this?
1
Not at all
2
3
4
5
Perfectly
52
Q

53- This patient is currently undergoing treatment with propranolol for a hemangioma. Which of the following tests or studies is most important to order prior to starting propranolol?

A. Laryngoscopy
B. Arterial studies
C. MRI of the brain
D. Thyroid stimulating hormone
E. Ophthalmologic exam

A

Correct choice: B. Arterial studies

Explanation: This is an example of a segmental hemangioma which is concerning for PHACES sysdrome. Patients with PHACES syndrome have arterial anomolies in the head/neck and heart. These anomolies in conjunction with propanolol may hypothetically put the patient at risk for arterial compromise/stroke, therefore it is important to check for any arterial abnormalities (MRA head and neck, echocardiogram) prior to starting propanolol in these patients. In addition to MRA of the head/neck and an echocardiogram, patients with PHACES syndrome should have an MRI brain to rule out posterior fossa malformation, and opthalmologic exam to look for eye abnormalities. However these tests/studies are not necessarily needed to start propnaolol. Patients with hemangiomas of the “bearded” region need direct visualization of the airway (laryngoscopy) when the hemangioma involves the bearded region to rule out airway compromise, however this is not necessarily needed prior to starting propanolol. Large hemagiomas can cause a consumptive hypothyroidism, therefore it is prudent to check a TSH.

How well did you know this?
1
Not at all
2
3
4
5
Perfectly
53
Q

54- An otherwise healthy child presents with the facial eruption seen below. Treatment with topical steroids did not help. What is the diagnosis?

A. Acne
B. Granulomatous perioral dermatitis
C. Tuberous sclerosus

D. Sarcoidosis
E. None of the above

A

Correct choice: B. Granulomatous perioral dermatitis

Explanation: The photo and histology are diagnostic of GPD. The photo shows a monomorphous perioral/periocular facial eruption. The histology shows granulomatous inflammation.

How well did you know this?
1
Not at all
2
3
4
5
Perfectly
54
Q

55- What should this patient be evaluated for?

A. Deafness
B. Cataracts
C. Neurocutaneous melanosis
D. Limb length discrepancy
E. Brachydactyly

A

Correct choice: C. Neurocutaneous melanosis

Explanation: This is a giant congenital melanocytic nevus which is associated with neurocutaneous melanosis. Brachydactyly is seen in Rubenstein-Taybi. Limb length discrepancy is seen in KlippelTrenaunay syndrome.

How well did you know this?
1
Not at all
2
3
4
5
Perfectly
55
Q

56- Which of the following is FALSE regarding the diagnosis in this child, which has been present since birth?

A. A ring of dark hair surrounding the cutaneous defect may be associated with an underlying neural tube defect.
B. It may be associated with in utero exposure to methimazole.
C. Most are an isolated abnormality.
D. If present bilaterally on the temples, it may be associated with leonine facies, upward slanting eyebrows and abnormal eyelashes.
E. If present on scalp with dominant dystrophic epidermolysis bullosa, it is known as Bart syndrome.

A

Correct choice: E. If present on scalp with dominant dystrophic epidermolysis bullosa, it is known as Bart syndrome.

Explanation: The correct answer is E (E is false). The photo depicts aplasia cutis congenita (ACC). Bart syndrome is ACC of the lower extremities (not scalp) and dominant dystrophic epidermolysis bullosa. Choice A describes the hair collar sign. Choice D describes Setleis syndrome.

How well did you know this?
1
Not at all
2
3
4
5
Perfectly
56
Q

57- A 9-year-old boy with developmental delay presents to the clinic with telangiectasias on the bulbar conjunctivae and neck. He has had gait difficulties and recurrent sinopulmonary infections since the age of 2 years. He is at risk for which of the following?

A. Epistaxis
B. Gastrointestinal bleeding
C. Lymphoma
D. CNS aneurysms
E. Pulmonary arteriovenous malformations

A

Correct choice: C. Lymphoma

Explanation: This case describes ataxia-telangiectasia syndrome, an autosomal recessive, multisystem disorder characterized by progressive neurologic impairment, cerebellar ataxia, variable immunodeficiency with susceptibility to sinopulomary infections, impaired organ maturation, X-ray hypersensitivity, ocular and cutaneous telangiectasias, and a predisposition to malignancy (particulary breast cancer, leukemia, and lymphoma - Choice 3). The features of epistaxis (Choice 1), gastrointestinal bleeding (Choice 2), CNS aneurysms (Choice 4), and pulmonary arteriovenous malformations (Choice 5) are seen in hereditary hemorrhagic telangiectasia (Osler-Weber-Rendu disease), which is due to a mutation in the ATM gene.

How well did you know this?
1
Not at all
2
3
4
5
Perfectly
57
Q

58- What is the best treatment of this condition?

A. Excision
B. Topical steroids
C. Penicillin
D. Phototherapy
E. 5-fluoruracil

A

Correct choice: C. Penicillin
Explanation: This is perianal strep which occurs in kids. Treatment is Penicillin or Amoxicillin for 10 days to three weeks. It can recur, and if it does it requires retreatment with antibiotics.

How well did you know this?
1
Not at all
2
3
4
5
Perfectly
58
Q

59- What is this often misdiagnosed as?

A. Sarcoma botryoides
B. Child abuse
C. Candidiasis
D. Normal anatomy

E. HSV

A

Correct choice: B. Child abuse

Explanation: This is lichen sclerosus et atrophicus in a child which happens in about 10% of the cases. It is frequently misdiagnosed as child abuse as there can be hemorrhagic bullae as part of the disease. The porcelain white and hemorrhagic bullae are clues to think of LS and A.

How well did you know this?
1
Not at all
2
3
4
5
Perfectly
59
Q

60- Hereditary coproporphyria is characterized with photosensitivity and the defect is:

A. Coproporphyrinogen oxidase
B. Porphobilinogen deaminase
C. Uroporphyrinogen III synthase
D. Uroporphyrinogen decarboxylase
E. ALA dehydratase

A

Correct choice: A. Coproporphyrinogen oxidase Explanation: Hereditary coproporphyria is characterized by photosensitivity and can also present with neuropsychiatric issues. They also have skin fragility and blistering is common. It is caused by a defect in Coproporphyrinogen oxidase. The other listed enzymes are not mutated in hereditary coproporphyria.

How well did you know this?
1
Not at all
2
3
4
5
Perfectly
60
Q

61- Which of the following is LEAST likely associated with this finding?

A. Renal abnormalities
B. Imperforate anus
C. Skin tag

D. Malformation of external genitalia
E. High output cardiac failure

A

Correct choice: E. High output cardiac failure

Explanation: The correct answer is E, which is a concern with diffuse neonatal hemangiomatosis. The remainder are within PELVIS syndrome (Perineal hemangioma, External genital malformation, Lipomyelomeningocele, Vesicorenal abnormalities, Imperforate anus, Skin tag).

How well did you know this?
1
Not at all
2
3
4
5
Perfectly
61
Q

62- Children with this disease are more likely to get what complication?

A. Lung disease
B. Malignancy
C. Calcinosis cutis
D. Coronary artery disease
E. Diabetes

A

Correct choice: C. Calcinosis cutis

Explanation: This is a picture of the ragged cuticles in DM. Children with dermatomyositis are more likely to have calcinosis cutis and vasculitis. They have do not have increased risk of malignancy. Interstitial lung disease is a rare complication of juvenile dermatomyositis.

How well did you know this?
1
Not at all
2
3
4
5
Perfectly
62
Q

63- A 1-month-old child has rapidly enlarging periorbital hemangiomas, what is the most effective treatment?

A. Systemic corticosteroids
B. Vincristine
C. Surgical excision
D. Oral propranolol
E. Follow up exam in 4 weeks

A

Correct choice: D. Oral propranolol

Explanation: Hemangiomas in the rapidly growing phase can be detrimental to vision if periorbital. Treatment is the standard of care with propranolol showing the greatest efficacy.

How well did you know this?
1
Not at all
2
3
4
5
Perfectly
63
Q

64- A 10-year-old girl presents with a swollen earlobe for several days treated by her pediatrician with oral antihistamines and topical mupirocin. With time, the eruption progressed to that shown in the image in addition to the development of fever, malaise, and neck pain. What would be the next best step in management for this patient?

A. Obtain a bacterial culture
B. Increase dosage and frequency of antihistamines and apply cool compresses
C. Order bloodwork for Lyme borreliosis
D. Treat with oral amoxicillin
E. Recommend acetaminophen for fever and apply mentholcontaining topicals for pruritus

A

Correct choice: D. Treat with oral amoxicillin

Explanation: Treating the patient with oral amoxicillin (Choice 4) is the correct answer. The image shows an example of a presentation of Lyme disease. Please note the targetoid nature of the eruption, which can sometimes be masked by hair on the scalp, delaying diagnosis and treatment.

Patients 8 years of age and younger should not receive treatment with doxycycline for Lyme as it is contraindicated due to risk of permanent tooth staining and dental enamel hypoplasia. Amoxicillin 250mg TID or 20mg/kg/day in TID dosing for at least two weeks is appropriate. Choice 1 is incorrect as a culture of the ear for bacteria would be indicated if there is evidence of otitis externa. The earlobe and conchal bowl do not have any surface disruption or drainage for culture. Choice 2 and 4 are incorrect as antihistamines, cool compresses, acetaminophen, and methol-containing products can be used for supportive treatment, but are not the most appropriate next step in this case. Choice 3 is incorrect because the patient has a characteristic erythema chronicum migrans eruption and symptoms consistent with early Lyme disease. Immediate treatment should be the next step. Additionally, only 25-50% of patients will have a positive Lyme ELISA screen when they present with erythema chronicum migrans.

How well did you know this?
1
Not at all
2
3
4
5
Perfectly
64
Q

65- You are called to the neonatal intensive care unit for evaluation of a newborn with congenital absence of skin on the lower extremities as illustrated in the figure. The most likely associated defect is:

A. Collagen IV
B. Collagen VII
C. BPAG1
D. BPAG2
E. Alpha-6-beta-4-integrin

A

Correct choice: B. Collagen VII

Explanation: Bart’s syndrome refers to congenital aplasia cutis (typically of the shins) and associated epidermolysis bullosa, most commonly dominant dystrophic EB. In dominant dystrophic EB there is a defect in Collagen VII. Bart’s syndrome is not associated with any of the other listed defects.

How well did you know this?
1
Not at all
2
3
4
5
Perfectly
65
Q

66- This baby does not have any systemic symptoms and these redbrown skin lesions heal on their own. What would this lesion stain positively with?

A. CD68
B. VVG
C. Thioflavin T
D. CD4
E. Langerin

A

Correct choice: E. Langerin

Explanation: This is Hashimoto-Pritzker disease, benign single system Langerhans cell histiocytosis. It is characterized by congenital selfhealing papulonodules that are red-brown color. It is composed of Langerhans cells which are langerin positive and CD1a positive and CD68 negative.

How well did you know this?
1
Not at all
2
3
4
5
Perfectly
66
Q

67- What condition is associated with this finding of inflammatory keratotic facial papules which may result in scarring and atrophy?

A. Chloracne
B. Systemic lupus erythematosus
C. Keratosis pilaris

D. Reiter’s syndrome
E. Ulerythema ophryogenes

A

Correct choice: E. Ulerythema ophryogenes

Explanation: Ulerythema ophryogenes is a rare disorder that affects children and young adults. It is characterized by keratosis pilaris atrophicans and loss of lateral third of eyebrow. The other listed conditions do not exhibit this

How well did you know this?
1
Not at all
2
3
4
5
Perfectly
67
Q

68- A middle-aged gentleman who avoided healthcare dies suddenly from gastrointestinal hemorrhage. Post-mortem examination reveals multiple soft blue compressible tumors on the trunk, arms, and tongue. Blue rubber bleb nevus syndrome is caused by a mutation in what gene?

A. TIE2
B. ENG
C. PIK3CA
D. PTEN
E. PTH/PTHrP type 1 receptor

A

Correct choice: A. TIE2

Explanation: Blue rubber bleb nevus syndrome is a rare sporadic, or sometimes autosomal dominant, disorder with soft compressible blue tumors on the trunk and arms. Nocturnal pain is characteristic. Gastrointestinal hemangiomas can cause hemorrhage or intussusception. Mutations in the TIE2 gene have been reported. It is also known as Bean Syndrome. ENG (endoglin, aka HHT1) is mutated in hereditary hemorrhagic telangiectasia (Osler-Weber-Rendu). PIK3CA mutations are associated with Klippel-Trenaunay syndrome, and CLOVES syndrome (and other overgrowth disorders). PTEN mutations are associated with Cowden, Bannayan-Riley-Ruvalcaba, and Proteus-like syndromes. Lastly, Mafucci syndrome is associated with a sporadic mutation in the PTH/PTHrP type 1 receptor.

How well did you know this?
1
Not at all
2
3
4
5
Perfectly
68
Q

69- what is the most likely diagnosis in an otherwise well appearing 3year-old child with a spontaneous eruption of the rash seen below?

A. Pityriasis rosea
B. Erythema dyschromicum perstans
C. Café-au-lait macules
D. Pigmentary mosaicism
E. None of the above

A

Correct choice: B. Erythema dyschromicum perstans

Explanation: The photo and histology are diagnostic of EDP, a benign condition characterized by sudden onset of blue-brown asymptomatic macules. Histology shows sparse perivascular infiltrate with melanophages.

How well did you know this?
1
Not at all
2
3
4
5
Perfectly
69
Q

70- Congenital Varicella Syndrome occurs after maternal varicella infection during which stage of pregnancy?

A. First 20 weeks
B. Third trimester
C. 5 days before and 2 days after delivery
D. 20-24 weeks
E. None of these answers are correct

A

Correct choice: A. First 20 weeks

Explanation: Congenital Varicella Syndrome occurs after maternal varicella-zoster virus infection early in pregnancy, most commonly between 8 and 20 weeks gestation. Characteristic findings of affected infants include some or all of the following: intrauterine growth resrtriction, cicatricial skin

lesions, ocular defects, limb abnormalities, CNS abnormalities. Newborns born to mothers who develop symptoms of VZV from 5 days before to 2 days after delivery are at risk of neonatal varicella.

How well did you know this?
1
Not at all
2
3
4
5
Perfectly
70
Q

71- Patients with congenital syphilis may have dental findings called:

A. Hutchinson’s teeth
B. Mulberry incisors
C. Erythrodontia
D. Odontogenic keratocysts
E. Jaw osteomas

A

Correct choice: A. Hutchinson’s teeth

Explanation: The dental findings that may be found in patients with late congenital syphilis are called Hutchinson’s teeth, which refers to enamel hypoplasia of maxillary central incisors and semi- lunar notching on the incisal edge of mandibular incisors. Mulberry molars (not incisors) may also been seen in patients with late congenital syphilis. Erythrodontia (red teeth) is seen in Congenital Erythropoietic Porphyria (aka Gunther’s disease). Odontogenic keratocysts may be seen in Gorlin syndrome (aka basal cell nevus syndrome). Jaw osteomas may be seen in Gardner syndrome.

How well did you know this?
1
Not at all
2
3
4
5
Perfectly
71
Q

72- A five year old male presents with lifelong mild to moderate asymptomatic enlargement of the right nipple compared to the left nipple; no gynecomastia present. What is the best approach to this situation?

A. Liposuction
B. Magnetic resonance imaging with and without contrast of the chest
C. Intralesional Kenalog injection
D. Reassurance
E. Plastic surgery referral

A

Correct choice: D. Reassurance

Explanation: Reassurance is all that is necessary for benign nipple enlargement. Patient can be referred to plastic surgery for revision if he chooses to do so in the future on his own accord.No intervention is necessary medically.

How well did you know this?
1
Not at all
2
3
4
5
Perfectly
72
Q

73- This 6-year-old girl is taken to the ED by her parents for fever for 5 days. You notice bilateral bulbar conjunctival injection without exudate and edema of the feet, but no lymphadenopathy. Her skin exam reveals scattered, dusky erythematous macules and patches, many of which were targetoid, on the torso and extremities. She also has the pictured oral finding. Which of the following is the most likely diagnosis?

A. Yersinia pseudotuberculosis
B. Kawasaki disease
C. Scarlet fever
D. Yellow fever
E. Toxic shock syndrome

A

Correct choice: B. Kawasaki disease

Explanation: The pictured oral finding is that of strawberry tongue. This patient fulfills clinical criteria for Kawasaki disease with fever for 5 days, bilateral bulbar conjunctival injection without exudate, strawberry tongue, edema of feet, and polymorphous exanthem. The remaining answer choices may also present with a strawberry tongue, but the clinical description best describes Kawasaki disease.

How well did you know this?
1
Not at all
2
3
4
5
Perfectly
73
Q

74- What is the diagnosis?
A. Eczema herpeticum

B. Verrucae
C. Molluscum contagiosum
D. Varicella
E. Impetigo

A

Correct choice: A. Eczema herpeticum

Explanation: Eczema herpeticum is a secondary infection that occurs in patients with atopic dermatitis. It is readily identified by monomorphic punched out erosions occuring within areas of eczema. Intact vesicles are less commonly seen. The treatment involves systemic antivirals as well as continuation of topical steroids. Bacterial and viral infectious are more commonly seen in patients with atopic dermatitis due to imparied skin barrier. Impetigo is caused by staph aureus and clinically characterized by honey crusting. Patients with atopic dermatitis are colonized with staph aurues and at risk of secondary impetiginization. Bleach baths are an effective component of maintenance treatment to prevent infection. Varicella, chickenpox, is characterized by lesions in various stages of evolution. It is caused by the varicella-zoster virus. Molluscum contagiosum is a benign and self limited cutaneous infection caused by the pox virus. The characteristic lesion is a dome shaped papule with central umbilication. Molluscum contagiosum can occur in areas of active atopic dermatitis. Verrucae are another benign and self limited viral infection caused by the human papilloma virus. They have a hyperkeratotic surface and thrombosed capillaries may be visible as black dots. They do not erode or ulcerate.

How well did you know this?
1
Not at all
2
3
4
5
Perfectly
74
Q

75- An 8 year old boy with seasonal allergies presents with red brown macules on the trunk present for several months. Parent mentions that when these lesions are scratched they form an irritated, red wheal. Which of the following sign is likely described?

A. Darier’s sign
B. Hutchinson sign
C. Auspitz sign

D. Asboe-Hansen sign
E. Homan’s sign

A

Correct choice: A. Darier’s sign

Explanation: Darier’s sign is seen in patients with urticarial pigmentosa, a common form of mastocytosis. Erythema and wheals are commonly elicited with stroking or rubbing secondary to mast cell degranulation. Symptoms can range from very mild (flushing, hives, no treatment needed) to life-threatening (vascular collapse). Asboe Hansen, commonly seen in pemphigus vulgaris, refers to extension of a bulla to adjacent unblistered skin when pressure is put on top of a bulla.

How well did you know this?
1
Not at all
2
3
4
5
Perfectly
75
Q

76- You are called to the NICU to evaluate this lesion, what is the next best step?

A. Fine Needle Aspiration
B. Acoustic testing
C. No further evaluation is necessary
D. MRI imaging
E. Surgical Excision

A

Correct choice: B. Acoustic testing

Explanation: This kodachrome shows a child with an accessory tragus. These typically occur on the preauricular cheek but can also occur on the lateral cheeks and neck. They can be associated with branchial arch syndromes (Goldenhar, oculo-auriculo-vertebral). They can be associated with hearing or genitourinary defects and therefore acoustic testing should be pursued. The other choices would not be the appropriate first step in evaluating accessory tragi.

How well did you know this?
1
Not at all
2
3
4
5
Perfectly
76
Q

77- A patient presents with a venous malformation. What is the most likely mutation?

A. GNAQ
B. TIE2/TEK2
C. Endoglin
D. GLUT1
E. Hamartin

A

Correct choice: B. TIE2/TEK2

Explanation:Venous malformations are compressible, rubbery, blue-red masses that are caused by a post zygotic somatic or germline mutation in TIE2/TEK2. This can be seen in the setting of Blue rubber bleb nevus syndrome or multiple cutaneous and mucosal venous syndrome. GLUT1 mutations occur in infantile hemangiomas. GNAQ mutations occur in Sturge-Weber syndrome and sporadic port wine stains. Endoglin mutations occur in hereditary hemorrhagic telangiectasia syndrome. Hamartin mutations occur in Tuberous Sclerosis.

How well did you know this?
1
Not at all
2
3
4
5
Perfectly
77
Q

78- What is the genetic mutation in hidrotic ectodermal dysplasia (HED)?

A. Plectin
B. Keratin 5
C. GJB6
D. Perforin
E. Filaggrin

A

Correct choice: C. GJB6

Explanation: Clouston syndrome (HED) is caused by mutations in the GJB6 gene (13q12), encoding the gap junction protein connexin 30 (Cx30).

How well did you know this?
1
Not at all
2
3
4
5
Perfectly
78
Q

79 -Late onset subungual keratotic tumors are associated with:

A. Incontinentia pigmenti

B. Neurofibromatosis Type 1
C. Carney complex
D. Cowden syndrome
E. Basal cell nevus syndrome

A

►A

A NEMO gene defect can cause subungual keratotic growths. The typical age of presentation is early adulthood.

How well did you know this?
1
Not at all
2
3
4
5
Perfectly
79
Q

80 -activating mutation in c-kit are found most often in which subset of patients with mastocytosis?

A. Adults with systemic disease refractory to imantinib
B. Adults with systemic disease associated with eosinophilia
C. Patients with familial history of mastocytosis
D. Adults with Telangiectasia Macularis Eruptiva Perstans
E. As a mosaic mutation in children with solitary cutaneous mastocytoma

A

►A

Almost all sporadic adult onset mastocytosis patients demonstrate mutations in c-kit. Most of these are activating mutations linked to the 816 codon. Unfortunately, patients with this specific mutation tend to not respond to systemic therapy with oral imantinib, a tyrosine kinase inhibitor.

How well did you know this?
1
Not at all
2
3
4
5
Perfectly
80
Q

81 -A 2-year old female presents with a skin eruption, and a history of joint swelling, painful movement and mucosal lesions. What is the most likely diagnosis?

A. Pustular psoriasis
B. PAPA syndrome
C. Familial Mediterranean fever
D. DIRA
E. Subcorneal pustular dermatosis

A

►D

Deficiency of the IL-1 receptor antagonist(DIRA). These patients, early in their lives, present with severe pustulosis and ichthyosiform skin eruptions. Also they can have joint pains and oral mucosal lesions. DIRA is a new addition to the spectrum of autoinflammatory disorders. It will not be encountered often, but its origin highlights the importance of IL-1 in skin function.

How well did you know this?
1
Not at all
2
3
4
5
Perfectly
81
Q

82 -The risk of fetal death with intrauterine parvovirus infection may occur with infection in which trimester:
A. First
B. Second
C. Third
D. First, Second and Third
E. None of these answers are correct

A

►D

Fetal hydrops may occur with parvovirus infection during all three trimesters although the greatest risk is during the second trimester. Congenital anomalies are not a feature.

82
Q

83 -This syndrome is associated with a large nevus sebaceous associated with ocular lesions, intracranial masses, mental retardation, seizures, and skeletal and pigmentary abnormalities:

A. Schimmelpenning’s syndrome
B. Linear epidermal nevus syndrome
C. Nevus sebaceous of Jadassohn
D. Epstein’s pearls
E. Pseudoverrucous syndrome

A

►A

Schimmelpenning’s syndrome involves a large nevus sebaceous associated with ocular lesions, intracranial masses and mental retardation. Nevus sebaceous is a congenital hairless, yellow to orange plaque on the scalp.

83
Q

84 -Which of the following disorders is more likely to occur in children with chronic fecal incontinence?

A. Langerhans cell histiocytosis
B. Perianal pseudoverrucous papules and nodules
C. Perianal streptococcal disease
D. Granuloma gluteale infantum
E. Seborrheic dermatitis

A

►B
The warty papules of perianal pseudoverrucous papules and nodules are seen more commonly in the setting of chronic fecal incontinence.

84
Q

85 -A newborn has a nodule over his lumbar spine. Skin biopsy reveals a lipoma. The next appropriate step is:

A. Observation
B. Excision of the lesion
C. Genetic testing
D. Imaging study
E. Malignancy work up

A

►D

The skin can provide an important clue to the presence of an underlying neural tube defect, such as meningomyelocele and encephalocele. Cutaneous lesions along the midline of the spine should always prompt consideration of this possibility. Although, midline neural tube defects are uncommon, early recognition and diagnosis of a spinal dysraphism can have important implications for early surgical correction and minimizing loss of neurologic function. Clues to the diagnosis

include a midline dimple, tuft of hair, lipoma, or vascular lesion. In these instances, imaging studies (MRI, CT, ultrasound) should be promptly initiated.

85
Q

86- An infant presents with the lesion depicted in the photo. What information do you provide the parents regarding this condition?

A. The lesion will go through rapid growth followed by stabilization and regression
B. The lesion will respond to laser therapy
C. The lesion will persist and may grow further
D. The lesion will resolve with antifungal therapy
E. The lesion is associated with a XO karyotype

A

►C

The lesion represented in the photo is an arteriovenous vascular malformation. These lesions do not typically regress.

86
Q

87- Which of the following diseases with immunodeficiency has an increased risk of lymphoreticular malignancy?

A. Wiskott-Aldrich syndrome
B. Chronic granulomatous disease
C. Job syndrome
D. Severe combined immunodeficiency syndrome
E. Leiner‟s disease

A

►A

Wiskott-Aldrich syndrome is an X-linked recessive disorder caused by mutations in WAS gene. Patients with Wiskott-Aldrich have atopic dermatitis with increased risks for secondary infection, thrombocytopenia, and recurrent bacterial infections. They are also at an increased risk for lymphoreticular malignancy (20%).

87
Q

88 - What is the most likely diagnosis is this 16 year old patient who developed generalized eruption 2 weeks after onset of sore throat due to strep infection?

A. Psoriasis
B. Pityriasis rosea
C. Cutaneous T-cell lymphoma
D. Contact dermatitis
E. Lichen planus

A

►A

Although the exact mechanism is not known, there is a well-known correlation between a history of upper respiratory infection secondary to group A beta-hemolytic streptococci and the subsequent development of guttate psoriasis. The eruption may resolve spontaneously or with treatment (topical steroids or ultraviolet therapy). Some portion of these patients eventually develop chronic, plaque- type psoriasis.

88
Q

89 -The differential diagnosis of zinc deficiency is least likely to include:

A. Granuloma gluteale infantum

B. Biotin deficiency
C. Multiple carboxylase deficiency
D. Cystic fibrosis
E. Holocarboxylase synthetase deficiency

A

►A

All of the options result in an eczematous acrodermatitis enteropathica-like eruption except granuloma gluteale infantum. As the name suggests, the lesions of granuloma gluteale infantum are granulomatous.

89
Q

90- Ankyloblepharon filiforme adnatum is seen with a defect in:

A. Plakophilin
B. Plakoglobin
C. Desmoglein
D. C-kit
E. p63

A

►E

The ectodermal-clefting syndromes are caused by a defect in the p63 gene. Specifically, AEC or Hay-Wells syndrome is comprised of ankyloblepharon filiforme adnatum, ectodermal dysplasia and clefting.

90
Q

91- A full term newborn develops erythematous, indurated plaques on the upper back. Which of the following tests should be performed?

A. Calcium
B. Echocardiogram
C. Blood culture
D. Thyroid function test
E. Alkaline phosphatase

A

►A

The most likely diagnosis is subcutaneous fat necrosis of the newborn. Hypercalcemia may result and serum calcium levels should be monitored up to 4 weeks after resolution of the skin.

91
Q

92- Rapp-Hodgkin is caused by a defect in the following gene:

A. Plakophilin
B. Desmoglein 1
C. Ectodysplasin A
D. P63
E. Connexin 30

A

►D

Rapp-Hodgkin, along with the other ectodermal dysplasia-clefting syndromes, is reportedly caused by a defect in the p63 gene.

92
Q

Slapped cheeks” followed by a lacy eruption on extremities:

A. ssDNA virus
B. dsDNA virus
C. ssRNA virus
D. dsRNA virus
E. Streptococcus

A

►A

The erythematous eruption on the cheeks precedes the generalized lacy rash of Fifth disease. The causative agent is parvovirus B19, a single stranded DNA virus.

93
Q

94- Variants of xeroderma pigmentosum are due to all of the following defects except:

A. Nucleotide excision repair
B. Helicase
C. Endonuclease
D. Thymidine kinase
E. Postrepliction repair

A

►D

The photosensitivity of xeroderma pigmentosum is caused by defect in DNA repair mechanisms. Thymidine kinase is not affected.

94
Q

95- Pastia”s lines are characteristic for which eruption:

A. Measles
B. Scarlet fever
C. Rubella
D. Kawasaki”s disease
E. Mumps

A

►B

Pastia”s lines are defined as a linear accentuation of the erythematous sandpaper rash within the flexures. This is classically described with scarlet fever.

95
Q

The genetic disorder depicted in this photo is caused by a mutation in:

A. MSH2
B. STK11
C. PTEN
D. Patched
E. TSC1

A

►B

The multiple labial lentigo seen on this young boy are seen with Peutz -Jeghers syndrome. A defect in the STK11 gene is responsible.

96
Q

97- Individuals with which of the following syndromes characteristically present with photosensitivity, mental retardation, a “wizened” appearance, “bird-headed” facies, and “Mickey Mouse” ears?

A. Tay Syndrome
B. Rothmund-Thomson Syndrome
C. Hutchinson-Gilford Progeria Syndrome
D. Cockayne Syndrome
E. Werner Syndrome

A

►D

Cockayne Syndrome is AR, caused by defective excision repair, cross-complementing group 8 gene (ERCC8). It presents with photosensitivity, mental retardation, and cachectic dwarfism. Patients have a characteristic “wizened” appearance, “bird-headed” facies, and “Mickey Mouse” ears. Cataracts, deafness, pigmentary retinopathy, dental caries, and skeletal, GU, and endocrine abnormalities may be seen.

97
Q

98- What syndrome is the disorder shown in the photo associated with?
A. Turner syndrome
B. Down syndrome
C. Noonan syndrome

D. Griscelli syndrome
E. Bloom syndrome

A

►C

The photo shows keratosis pilaris atrophicans faceii and surgically corrected ptosis. Both of these findings are associated with Noonan syndrome.

98
Q

99- Juvenile self-healing papular mucinosis is associated with which of the findings?

A. Arthralgias
B. Cataracts
C. Type I diabetes
D. Calcinosis cutis
E. Xeropthalmia

A

►A

Self-healing papular mucinosis is a rare disorder characterized by small, nontender, flesh-colored papules commonly seen on the head, neck, trunk, and periarticular areas of children. Self-healing papular mucinosis is associated with systemic findings during acute stages, including fever, arthralgias, weakness, and often carpal tunnel syndrome. A handful of cases have been reported in adults, although systemic symptoms are not typically found in this population.

99
Q

100- An 8 year-old boy presents with pink, flushed cheeks and a low-grade fever. One week later, a lacy eruption appears. What is the most likely etiology?

A. Paramyxovirus
B. Togavirus
C. Group A streptococcus
D. HHV6
E. Parvovirus

A

►E

Fifth disease is caused by Parvovirus B19. Most cases start with prodrome of fever, malaise, headache and rhinorrhea. Cutaneous reaction follows approximately 5-7 days later with erythema of the cheeks (“slapped cheeks”) and reticulate rash of the trunk and extremities.

100
Q

101 -An infant presents with yellowish-brown, crusted papules with petechiae in a seborrheic distribution. Which of the following statements about this entity is likely to be true?

A. This presentation is consistent with Jacquet’s dermatitis
B. CD1+, S100+ cells with comma-shaped nuclei should be seen on biopsy
C. This presentation occurs when the infant is weaned off of breast milk
D. An autosomal recessive defect in holocarboxylase synthetase is the cause
E. Maternal-fetal transmission most likely occured in the peripartum period

A

►B

This infant has Langerhans Cell Histiocytosis (Letterer-Siwe disease). Multisystem involvement may be present. Jacquet’s Erosive Dermatitis presents with severe erosive papules in the diaper region, and is multifactorial in etiology (yeast, irritant dermatitis, moisture). Acrodermatitis enteropathica (zinc deficiency) presents with brown, orange crusted plaques with vesicles and bullae, especially in perineal and perioral areas and distal extremities. The inherited form occurs when the infant is weaned off of breast milk. Biotin deficiency presents similarly; the neonatal form is attributed to an AR defect in holocarboxylase synthetase.

101
Q

102- The standard of care of patients with acute Kawasaki”s disease is:

A. Prednisone
B. Supportive care
C. Aspirin and IVIG
D. Penicillin
E. Acetaminophen and IVIG

A

►C

Kawasaki disease, also called mucocutaneous lymph node syndrome, is an acute febrile disorder based on the clinical criteria of changes in peripheral extremities, polymorphous exanthema, conjunctival injection without exudates, changes in the lips or oral cavity, acute cervical lymphadenopathy. Fever must be present, lasting more than 5 days. Treatment is aimed to prevent coronary aneurysms and myocardial infarction. Treatment for acute Kawasaki disease is intravenous immunoglobulin 2 g/kg over 10-12 hours and aspirin therapy.

102
Q

103- Which of the following is a ssDNA virus:

A. Herpesvirus
B. Parvovirus
C. Picornovirus
D. Adenovirus
E. Parapox

A

►B

Parvovirus is the only ssDNA virus listed.

103
Q

104 -Which of the following is the most common long term sequelae from congenital rubella syndrome?

A. Macrocephaly

B. Saber shins
C. Deafness
D. Nystagmus
E. Microcephaly

A

►C

Deafness may occur in up to 50% of infants with congenital rubella syndrome.

104
Q

105- Osteopathia striata is found in which disorder?

A. McCune-Albright syndrome
B. Neurofibromatosis I
C. Buschke-Ollendorff syndrome
D. Gorlin’s syndrome
E. Focal dermal hypoplasia

A

►E

Osteopathia striata (vertical striations in the metaphysis of long bones on x-ray) is seen is greater than 80% of cases of focal dermal hypoplasia (or Goltz syndrome). Polyostotic fibrous dysplasia with recurrent fractures is seen in McCune-Albright syndrome. Sphenoid wing dysplasia and thinning of long bone cortex is found in neurofibromatosis I. Osteopoikilosis is an asymptomatic x- ray finding in patients with Buschke-Ollendorf syndrome. Osteopoikilosis reflects ectopic calcification that does not increase risk of fracture. Bifid ribs, vertebral fusion/Sprengel deformity of the spine, and kyphoscoliosis can be seen in basal cell nevus syndrome (Gorlin’s syndrome).

105
Q

106- What is the most common cause of neonatal purpura fulminans?

A. Strep
B. Staph
C. Varicella
D. Protein C deficiency

E. Factor V Leiden deficiency

A

►D

In children, purpura fulminans may have several causes. It is a highly characteristic feature of meningococcal septicemia and may occur as a sequel to a number of infections, including streptococcal infections, varicella, and measles. In the neonate, however, its occurrence is highly suggestive of homozygous protein C deficiency.

106
Q

107 -A newborn infant presents with ring of long, dark, coarse hair surrounding a midline scalp patch of alopecia. What is associated with this finding?

A. Alopecia areata
B. Ectopic brain tissue
C. Thyroid disease
D. Nevus sebaceus of Jadassohn
E. Deafness

A

►B

The “hair collar sign” is associated with ectopic brain tissue and is thought to arise from a congenital herniation through the skull. Caution must be used in evaluation as biopsy or needle aspiration may lead to retrograde infection.

107
Q

What is the most likely diagnosis?

A. Psoriasis
B. Langerhans cell histiocytosis
C. Granuloma gluteale infantum
D. Contact dermatitis
E. Perianal streptococcal disease

A

►E

The bright red erythema of perianal streptococcal disease can also involve the creases of the groin and axillae.

108
Q

The muscle that is affected in a patient with a fibromatosis colli is:

A. Sternocleidomastoid muscle
B. Masseter muscle
C. Frontalis muscle
D. Bicepts
E. Tricepts

A

►A

Fibromatosis colli affects the lower sternocleidomastoid muscle that my arise from birth trauma. These occur in the pediatric patients and can spontaneous remit.

109
Q

110- Regarding the Langerhans cell histiocytoses, which of the following statements is true:

A. Letterer-Siwe primarily affects children ages 2-4 years
B. Hand-Schuller-Christian represents the triad of diabetes mellitus, bone lesions, and exophthalmos
C. Hashimoto-Pritzker is characterized by a solitary, self-resolving papule or nodule
D. Approximately 50% of cells will have Birbeck granules.
E. Langerhans cell histiocytoses stain positively for CD1a, S100, HAM56, and CD68.

A

►D

Letterer-Siwe primarily affects children under age 2. Hand-Schuller-Christian represents the triad of diabetes insipidus, bone lesions, and exophthalmos. Hashimoto-Pritzker is characterized by a self- resolving widespread eruption. Langerhans cell histiocytoses stain positively for CD1a and S100 but not for HAM56 and CD68.

110
Q

111- A 2 day-old full term neonate develops blotchy erythematous macules with small central pustules over the upper trunk and extremities. A gram stain reveals predominantly eosinophils. What is the most likely diagnosis?

A. Erythema toxicum neonatorum
B. Incontinentia pigmenti
C. Urticaria pigmentosa
D. Transient neonatal pustular melanosis
E. Miliaria

A

►A
Erythema toxicum neonatorum is a very common eruption in healthy newborns. A gram stain reveals sterile pustules containing eosinophils.

111
Q

112- Which of the following is a sign of EARLY postnatal congenital syphilis?

A. Clutton’s joints
B. Higoumenaki’s sign
C. Hutchinson’s teeth

D. Saber shins
E. Wimberger’s sign

A

►E

Wimberger’s sign is a sign of early postnatal congenital syphilis (occurs < 2 years old). All other choices are signs of late postnatal congenital syphilis. Wimberger’s sign is a radiographic sign showing a sawtooth appearance of the proximal tibia. Clutton’s joints are nontender and represent synovitis with effusions of the knees and elbows. Higoumenaki’s sign is a unilateral clavicular enlargement secondary to periostitis. Hutchinson’s teeth are centrally notched, wide spaced, peg shaped upper incisors. Saber shins are anterior bowing of the tibia.

112
Q

113- Which of the following bullous disorders is due to a target antigen that is a 97 kD protein which is a member of BPAG-2?

A. Bullous Dermolysis of the Newborn
B. Neonatal Pemphigus
C. Linear IgA Bullous Dermatosis
D. Junctional epidermolysis bullosa (Herlitz)
E. Epidermolysis bullosa simplex

A

►C

Linear IgA Bullous Dermatosis is due to a target antigen that is a 97 kD protein which is a member of BPAG-2. Transient bullous dermolysis of the newborn is due to a transient defect in intracytoplasmic packaging or in the transport of type VII collagen within basal keratinocytes. Neonatal pemphigus is due to maternal transfer of IgG antibodies reacting to Desmoglein 3 (pemphigus vulgaris). Epidermolysis bullosa simplex is due to keratin 5/14 abnormalities and junctional EB (Herlitz) is due to defects in Laminin 5.

113
Q

114 -The most common age group for papular-purpuric gloves and socks syndrome is:

A. Newborns

B. Toddlers
C. 6-10 year olds
D. Adolescents
E. Elderly

A

►D

This unique presentation of parvovirus infection typically occurs in adolescents and young adults.

114
Q

115 -What the most likely diagnosis?

A. Atopic dermatitis
B. Lamellar ichthyosis
C. Rud syndrome
D. Wiskott-Aldrich syndrome
E. Keratosis pilaris

A

►A

Ichythosis vulgaris and hyperlinear palms are both independent minor criteria for the diagnosis of atopic dermatitis in both children and infants.

115
Q

116 -In a child with zinc deficiency, yet normal or near normal zinc levels, which test could be a valuable adjunctive test?

A. Magnesium
B. Niacin
C. Manganese
D. Alkaline phosphatase
E. Iron

A

►D

The diagnosis of zinc deficiency should be consideredi in at-risk individuals with acral or periorificial dermatitis. Chronic diaper dermatitis in an infant should lead to the evaluation for zinc deficiency. A low serum zinc level can usually confirm the diagnosis. If normal or near normal a low serum alkaline phosphatase, a zinc-dependent enzyme, may be a valuable adjunctive test.

116
Q

117- A 6 month-old presents with orange-brown crusted plaques around the mouth and groin. Several bullae are present on the fingers and toes. Which of the following laboratory values is likely to be abnormal?

A. Hematocrit
B. Calcium
C. Platelet count
D. ALT
E. Alkaline phosphatase

A

►E

The most likely diagnosis is acrodermatitis enteropathica. Alkaline phosphatase is a zinc dependant enzyme that is decreased in response to low serum zinc levels.

117
Q

118- What is the best therapeutic option?

A. Oral cephalexin
B. Topical immune modulator
C. Oral acyclovir

D. Topical mupirocin
E. Oral prednisone

A

►C

The condition shown in an example of eczema herpeticum, also called Kaposi varicelliform eruption. It occurs when pre-existing dermatitis becomes superinfected with a viral infection, most commonly atopic herpes infection of atopic dermatitis. The initial treatment of choice is antiviral medications.Oral acyclovir is accpetable if the patient is able to swallow and is not clinically deteriorating, otherwise IV acyclovir is necessary.

118
Q
  1. A- young boy presents with a port-wine stain, a Mongolian spot, and a nevus spilus. Which phakomatosis pigmentovascularis does he have?

A. Type I
B. Type II
C. Type III
D. Type IV
E. Type V

A

►D

This patient has type IV phakomatosis pigmentovascularis characterized by a port-wine stain, Mongolian spot, and nevus spilus. All types have a nevus flammeus. In addition, type II demonstrates a Mongolian spot (may be associated with granular cell tu mor), type III a nevus spilus, and type V cutis marmorata telangiectasia. Type II-IV can also have a nevus anemicus. Phakomatosis pigmentovascularis is thought to be caused by the “twin spot” phenomenon. Type I:

CM + epidermal nevus Type II: CM + dermal melanocytosis +/- nevus anemicus Type III: CM + nevus spilus +/- nevus anemicus Type IV: CM + dermal melanocytosis + nevus spilus +/ nevus anemicus. (ie types II+III) Type V: CM + cutis marmorata

119
Q

120 -The association of Port-wine stains on a limb with soft tissue swelling with or without bony over growth is:

A. Bannayan-Riley-Ruvalcaba syndrome
B. Goldenhar’s syndrome
C. Sturge-Weber syndrome
D. Klippel-Trenaunay syndrome
E. Proteous syndrome

A

►D

The association of port-wine stain on a limb with soft tissue swelling with or without bony overgrowth is Klippel-Trenuanay syndrome. Klippel-Trenaunay syndrome is characterized by the Triad of port-wine malformations in association with deep venous system malformations, superficial varicosities, and bony and soft tissue hypertrophy. Sturge-Weber syndrome has 2 essential components: Facial port-wine stain and homolateral leptomeningeal angiiomattosis. The port wine stain most commonly involves the areas innervated by the ophthalmic(V1) and maxillary (V2) divisions of the trigeminal nerve. Complications of leptomeningeal angiomatosis are epilepsy, mental retardation, and occasionally, contralateral hemiplegia. Proteus Syndrome is characterized by vascular malformations including nevus flammeus, hemihypertrophy, macrodactyly, verrucous epidermal nevus, soft-tissue subcutaneous masses, and cerebriform overgrowth of the plantar surface. Bannayan Riley Ruvalcaba syndrome may include multiple cutaneous and visceral venous, capillary, and lympathtic malformations, macroephaly, pseudopapilledema, systemic lipoangiomatosis, spotted pigmentation of the penis, hamartomatous intestinal polyps, and rarely trichilemmonmas. (multiple subcutaneous lipomas as well as acanthosis nigricans).

120
Q

121- Rhinorrhea, condylomata lata, and mucous patches are all seen with which congenital disorder?
A. Rubella
B. Toxoplasmosis

C. Herpes simplex virus
D. Syphilis
E. Human papillomavirus infection

A

►D

Signs of congenital syphilis include rhinorrhea, snuffles, rhagades, condylomata lata, and mucous patches. Condylomata lata or a generalized papulosquamous eruption of secondary syphilis may be present in diaper area.

121
Q

122- Psammomatous melanotic schwannomas are associated with:

A. Bloom syndrome
B. Carney complex
C. Neurofibromatosis Type 1
D. Neurofibromatosis Type 2
E. Tuberous sclerosis

A

►B
Psammomatous melanotic schwannomas have been described in the Carney complex, a defect in the tumor suppressor gene, PRKAR1A.

122
Q

123- Neonatal acne is associated with species of which organism:

A. Staphylococcus
B. Propionibacterium

C. Candida
D. Malassezia
E. Streptococcus

A

►D

Acne which develops within the first 30 days of life is termed neonatal acne. Neonatal acne has a predilection for the face, chest, back and groin appearing as small, discrete papules at 2 to 4 weeks of age, and persisting for up to 8 months. As these lesions are self-resolving, no treatment is necessary, though 2.5% benzoyl peroxide may hasten resolution. Neonatal acne is quite common and is postulated to occur as a result of hyperplasia of premature sebaceous glands coupled with transient increases in circulating androgens. More recent data suggests that Malassezia species may be implicated at etiologic factors in neonatal acne therefore ketoconazole is a treatment. These organisms have been cultured from the skin of affected patients, though their exact role in unclear.

123
Q

124- An infant presents with the lesion depicted in the photo. Which of the following is possible?

A. Sternal clefting
B. Supraumbilical raphe
C. Dandy-Walker malformation
D. All are possible
E. Congenital cataracts

A

►D

Extensive facial hemangiomas are a component of the PHACES syndrome.

124
Q

125- What is the diagnosis?

A. Transient neonatal pustular melanosis
B. Acropustulosis of infancy
C. Neonatal cphalic pustulosis
D. Miliaria
E. Erythema toxicum neonatorum

A

►C

In neonatal cephalic pustulosis (neonatal acne), papules and pustules are present but comedones are absent. Spontaneous remission occurs.

125
Q

126 -What is the most common tumor associated with this condition?

A. Trichoblastoma
B. Syringocystadenoma papilliferum
C. Sebaceous carcinoma
D. Basal cell carcinoma
E. Trichoadenoma

A

►A

Recent reviews have identified trichoblastoma as the most common tumor arising within nevus sebaceous.

126
Q

127 -A 10 year old girl presents with desquamation of the fingertips. Which exam should be ordered?

A. Renal ultrasound
B. ASO titer
C. EEG
D. Eye exam
E. Chest x-ray

A

►B

Desquamation of the fingertips is commonly associated with infections with group A betahemolytic Strep and Staph aureus, including scarlet fever, perianal Strep, Staph scalded skin syndrome, and toxic shock syndrome. Thus ASO titer would be indicated in this case. Fingertip desquamation is also a manifestation of Kawasaki disease.

127
Q

128- Menkes kinky hair syndrome is associated most commonly with:

A. Trichorrhexsis invaginata
B. Trichostasis spinulosa
C. Pili multigemini
D. Pili torti
E. Plica neuropathica

A

►D

The X-linked recessive Menkes kinky hair syndrome is associated with multiple hair shaft abnormalities, most characteristically, pili torti.

128
Q

129 -Which of the following syndromes is associated with development of giant cell epulides?

A. Cherubism
B. Gardner syndrome
C. Cowden syndrome
D. Multiple hamartoma syndrome
E. CHILD syndrome

A

►A

Cherubism is an autosomally-dominant inherited disorder characterized by asymptomatic fibro- osseous hyperplasia and replacement of normal bone which commences in childhood. Typically the mandible is affected, leading to progressive enlargement and swollen “cherub-like” cheeks. Similar involvement of maxillary bones results in periorbital enlargement, resulting in the tendency of affected individuals to “look up towards the sky.” Frequently the disorder is self- limited and self- regressive. Mutations in SH3BP2 have been identified as a candidate gene defect. Affected individuals may also present with single or multiple giant cell epulides (singular: epulis), which is a benign, bluish-red nodule arising on the gingiva, near deciduous incisors or bicuspids. It bears histologic similarity to giant cell tumor of the tendon sheath.

129
Q

Which of the following is not a major criterion for Kawasaki‟s disease:

A. Fever >5 days
B. Palmoplantar erythema > desquamation
C. Cardiac aneurysm
D. Strawberry tongue/ red lips
E. Cervical adenopathy

A

C

Cardiac aneurysm is a serious complication of Kawasaki‟s disease. However, as the cardiovascular manifestations generally present 1 –5 months after presentation, they are not criteria for diagnosis.

130
Q

131 -What is the function of the gene which is defective in ataxia-telangiectasia?

A. Gap junction protein
B. Cross-linking of structural proteins in the protein and lipid envelope of the upper epidermis
C. Pathway of cholesterol biosynthesis
D. DNA repair protein
E. Tumor supressor protein

A

►D

The defective gene is ataxia-telangiectasia (Louis-Bar syndrome) is the ATM gene, which is responsible for DNA repair, especially after ionizing radiation. Tumor suppressor genes mutations are responsible for basal cell nevus syndrome, xeroderma pigmentosum, Muir-Torre syndrome, dyskeratosis congenita, Gardner syndrome, Peutz-Jeghers syndrome, Cowden syndrome, and MEN syndromes. Connexins are gap junction proteins that are responsible for intercellular communication and signaling. Mutations in connexins are responsible for Vohwinkel syndrome and erythrokeratoderma variabilis .Mutations in the cholesterol biosynthesis pathways cause CHILD syndrome and Conradi -Hunermann syndrome. Tranglutaminase 1 (TGM 1) is involved in the normal cross-linking of structural proteins in the protein and lipid envelope of the upper epidermis. TGM 1 is mutated in lamellar ichthyosis and congenital ichthyosiform erythroderma.

131
Q

What is the most likely diagnosis?
A. Traction alopecia
B. Tinea capitis
C. Aplasia cutis congenita
D. Nevus sebaceous
E. Alopecia areata

A

►C

Aplasia cutis congenita is characterized by the absence of a portion of skin, most commonly presenting as a solitary defect on the scalp, but sometimes it may occur as multiple lesions. The lesions are non-inflammatory and well demarcated appear as an atrophic, membranous, ulcerated area with alopecia. The condition may be associated with other physical anomlies.

132
Q

Which of the following is the most common complication associated with cutis marmorata telangectatica congenita?

A. Seizure disorder
B. Atrial septal defect
C. Systemic lupus erythematosus
D. Hypercalcemia
E. Limb hypertrophy or atrophy

A

►E

Hypertrophy or atrophy of the affected limb is the most likely consequence of cutis marmorata telangectatica congenita. Orthopedic evaluation should be a part of the patient‟s routine management.

133
Q

What is the most likely diagnosis?

A. Psoriasis
B. Pityriasis rosea
C. Cutaneous T-cell lymphoma
D. Contact dermatitis
E. Lichen planus

A

►D

This periumbilical eruption is classic for a contact nickel allergy. The metal snaps on pants are the cause in this case.

134
Q

135 -A 6-year-old boy presents with osteoma cutis on his face. Which of the following genes is most likely defective?

A. GNAS
B. NSDHL
C. PTEN
D. SPINK5
E. SLURP1

A

►A

Osteoma cutis in a child is associated with Albright‟s hereditary osteodystrophy (pseudohypoparathyroidism), which is caused by a mutation in GNAS1. Albright hereditary osteodystrophy is characterized by including short stature, obesity, round facies, subcutaneous ossifications, brachydactyly, and other skeletal anomalies. Some patients have mental retardation. AHO is often associated with pseudohypoparathyoidism, hypocalcemia, and elevated PTH levels (PHP Ia) (OMIM, 103580). NSDHL is defective in CHILD syndrome, PTEN in Cowden‟s syndrome, SPINK5 in Netherton‟s syndrome and SLURP1 in Mal de Maleda.

135
Q

136 -Most common location of cutaneous lesions in neonatal lupus erythrematosus is
A. Cheeks
B. Nose
C. Scalp
D. Perioral
E. Periorbital

A

►E

neonatal lupus erythrematosus(NLE) is a form of subacute cutaneous lupus erythrematosus(SCLE) that occur in infants whose mothers have anti-Ro autoantibodies. Unlike SCLE in adults, lesions of

NLE have a predilection for the face especially periorbital region. Other sites include scalp, arms, legs, trunk and groin.

136
Q
  • An infant who presents with this abnormality will often have the following:

A. The lesion will be on the left-side in a segmental distribution
B. Airway restriction
C. Have multiple liver hemangiomas
D. Most likely be a male infant
E. An anterior fossa malformation

A

►A

PHACES syndrome represents posterior fossa malformations, hemangiomas, arterial abnormalities, cardiac abnormalities, eye abnormalities, and sternal defects. Facial hemangiomas are most often on the left-side of the face. The majority of reported cases are female.

137
Q

138 -Subcutaneous fat necrosis of the newborn is usually treated with:

A. Aspirin
B. Topical calcipitriol
C. Excision
D. Observation
E. Retinoids

A

►D

Subcutaneous fat necrosis of the newborn is rare and self-limited. It is thought to result from perinatal stress and resolves in 3-6 months. Treatment is usually not necessary unless hypercalcemia develops.

138
Q

139 -A 10 year-old male presents with new pink-red papules distributed on his bilateral cheeks, elbows, knees, and buttocks. The lesions are not pruritic. His mother notes he has suffered from low grade fevers over the past few days, but is otherwise healthy. Which of the following is the most likely diagnosis?

A. rubella
B. rubeola
C. roseola
D. papular acrodermatitis of childhood
E. dermatitis herpetiformis

A

►D

Papular acrodermatitis of childhood (Gianotti-Crosti syndrome) typically affects children 6 months to 10 years of age, and is characterized by symmetric papules (and sometimes papulovesicles) involving the face and buttocks (“four cheeks”) and extensor surfaces of the upper and lower extremities. Lesions tend to be asymptomatic. Multiple infectious etiologies have been implicated, including Epstein-Barr virus (most common in the U.S.) and hepatitis B (most common in Europe). However, the eruption is most often self-limited, resolving in 4 to 8 weeks, and thus further work- up (e.g. testing for hepatitis) is indicated only if additional clinical symptoms are present.

139
Q

140- Hemangiomas of infancy are more likely to occur:

A. In males
B. In Asians
C. In children who are large for gestational age
D. In multiple gestation
E. In post term infants

A

►D

Hemangiomas of infancy occur more commonly in girls, in Caucasians, in babies who are premature and low birth weight. They are more likely to occur in multiple gestation

140
Q

141- Which presentation of psoriasis is more common in children

A. Pustular psoriasis
B. Acrodermatitis continua of Hallopeau
C. Keratoderma blennorragica
D. Erythrodermic psoriasis
E. Guttate psoriasis

A

►E

The majority of cases of guttate psoriasis occur in persons under the age of 30.

141
Q

142- A child presents with high fever, strawberry tongue, lymphadenopathy, and a polymorphous exanthem. Approximately how many of these patients will have coronary artery aneurysms if left untreated?

A. Two percent
B. Five percent
C. Ten percent
D. Twenty-five percent
E. Sixty percent

A

►D

This child has Kawasaki’s disease, or mucocutaneous lymph node syndrome. Other features include conjunctival injection and extremity erythema and edema. Twenty-five percent of untreated patients go on to develop coronary artery aneurysms. Treatment is with IVIG and aspirin.

142
Q

143- What syndrome can accessory tragi be associated with?

A. Goldenhar syndrome
B. Turner syndrome
C. Neurofibromatosis
D. Ichthyosis
E. Birt Hogg Dube

A

►A
Accessory tragus is the most common congenital defect of the external ear and linked to maldevelopment of the first branchial arch. It can be associated with several syndromes including Goldenhar syndrome, in which epibulbar dermoid and vertebral defects are also common. An accessory tragus usually it appears as a small skin-colored tag or nodule arising near the tragus; it is composed of normal epidermis with dermal adipose tissue, pilosebaceous units, eccrine glands, elastic fibers, and cartilage.

143
Q

-Which of the following most commonly presents as collodion baby?

A. Ichthyosis vulgaris
B. X-linked ichthyosis
C. Lamellar ichthyosis
D. Bullous congenital ichthyosiform erythroderma
E. Sjogren-Larsson syndrome

A

►C

The most common presentation of collodion baby is lamellar ichthyosis, followed by congenital ichthyosiform erythroderma. Patients with ichthyosis vulgaris and x-linked ichthyosis are normal at birth. Bullous congenital ichthyosiform erythroderma or epidermolytic hyperkeratosis presents with

widespread bullae, erthroderma, and denuded skin. Sjogren-Larsson presents with generalized ichthyosis and erythroderma in infancy. It is important to know the at-birth presentations of all the disorders of cornification.

144
Q

145- Multiple cylindromas are associated with:

A. Myotonic dystrophy
B. Cowden syndrome
C. Carney complex
D. Trichoepitheliomas
E. Pilomatrichomas

A

►D

The Brooke-Spigler syndrome is defined by the presence of multiple trichoepitheliomas and cylindromas.

145
Q

146 -What is the most likely diagnosis?

A. Psoriasis
B. Langerhans cell histiocytosis
C. Seborrheic dermatitis
D. Contact dermatitis
E. Perianal streptococcal disease

A

►A

The well-defined erythematous plaque is an example of psorisis in the diaper area. The moist nature of the diaper environment results in a loss of the classic silvery scale.

146
Q

147 -A 12 y/o female with autoimmune hepatitis on oral prednisone presents with an eruption on the face for 3 weeks. What is the most likely diagnosis?
A. Tinea facei
B. Acne vulgaris
C. Keratosis pilaris
D. Pityrosporum Folliculitis
E. Diagnosis not listed

A

►D

Pityrosporum Folliculitis is often associated with oral corticosteroid use such as prednisone, diabetes and other immune suppressed states. It presents as 1-2mm pruritic monomorphic papules and pustules on chest, back, upper arms, sometimes face.

147
Q

148- The causative agent of Roseola is:

A. A ssDNA virus
B. A dsDNA virus
C. A ssRNA virus
D. A dsRNA virus
E. Streptococcus

A

►B

Roseola is caused by Human Herpesvirus 6, a double stranded DNA virus.

148
Q

149- A young girl presents with enlarged tongue, exomphalmos, and organomegaly and has history of Wilm’s tumor. What cutaneous finding is most likely on physical exam?

A. Palmoplantar keratoderma
B. Lymphatic malformation
C. Acral edema
D. Midline capillary malformation
E. Angiokeratoma

A

►D

This patient has Beckwith-Wiedemann syndrome, a sporadic condition that is also known as exomphalos-macroglossia-gigantism syndrome. It also features a midline capillary malformation, linear earlobe creases, intestinal malrotation, and other tumors such as rhabdomyosarcoma an hepatoblastoma.

149
Q

150 -A 2 week-old infant is brought to the ER with a rash on her face. She is found to have a 3rd degree heart block. What is the risk that a second child born to this mother will have the same diagnosis?

A. 5%
B. 10%
C. 25%
D. 50%
E. 100%

A

►C

The diagnosis here is neonatal lupus erythematosus. Babies are normal at birth and develop skin lesions within the few months of life. About half of these babies will have an associated congenital heart block, usually 3rd degree, which is permanent. Most infants with NLE are girls and are born to mothers who are Ro/La positive. There is a 25% chance that a second child born to the same mother will have NLE.

150
Q

151- Which enanthem is most commonly seen in association with Exanthem subitum?

A. Koplik spots

B. Red strawberry tongue
C. Chapped lips; dry, red mucosa
D. Red macules and streaks on the soft palate
E. Palatal erosions

A

►D

Exanthem subitum (roseola or Sixth disease) is caused by HHV6, a dsDNA virus. It presents with high fever for several days followed by an exanthem of erythematous macules and papules on the trunk that begins as the fever ends. An associated enanthem of red macules/streaks on the soft palate may be seen. Koplik spots are seen in measles; the red strawberry tongue (following the white strawberry tongue) is seen in Scarlet fever; chapped lips and dry, red mucosa may be seen in Kawasaki’s disease; palatal erosions may be seen in Papular-purpuric gloves and socks syndrome.

151
Q

152 -An infant presents with red-purple, granulomatous nodules occurring in the diaper area. The etiology is secondary to local irritation, maceration and Candida albicans. What is the most likely diagnosis?

A. Granuloma gluteale infantum
B. Langerhans cell histiocytosisc.
C. Seborrheic dermatitis
D. Biotin deficiency
E. Psoriasis

A

►A

The etiology of granuloma gluteale infantum is multifactorial, resulting from the unique environment of the diaper area. Treatment consists of topical antifungal agents, barrier creams, and anti-inflammatory agents as needed.

152
Q

153- What is the diagnosis?

A. Unilateral laterothoracic exanthem

B. Roseola
C. Rubella
D. Urticaria
E. Contact dermatitis

A

►A

The child in the photo depicts the “Statue of Liberty” sign of unilateral laterothoracic exanthem.

153
Q

154- An infant with failure to thrive has multiple xanthomas on skin exam and foamy histiocytes on bone marrow biopsy. Your diagnosis is:

A. Gauchers disease
B. Tay-Sacs disease
C. Niemann-Pick disease
D. Fabris disease
E. Hunters syndrome

A

►C

Niemann-Pick disease is an autosomal recessive disease caused by mutations in sphingomyelin phosphodiesterase-1. Patients with Type A Niemann-Pick disease have onset in infancy and die by 3 may have xanthomas, progressive psychomotor deterioration, hepatosplenomegaly, blindness, cherry red spots, and deafness. Type B is the late onset non-neurologic form that only involves the viscera and patients survive into adulthood.

154
Q

155- A 3 month old baby girl is brought to your office for consultation. You note an infantile hemangioma on the tip of the nose. Imaging reveals no underlying structural anomalies in the cerebro-vasculature. You discuss starting propranolol. What side effect do you need to monitor for?

A. Hypertension
B. Hypercalcemia
C. Hypoglycemia
D. Tachycardia
E. Hypernatremia

A

►C

In the treatment of infantile hemangiomas with propranolol, the most common serious adverse effects are bradycardia and hypotension. Other potential adverse effects include bronchospasm and hypoglycemia. Sustained hypoglycemia in infancy has been associated with longterm neurologic sequelae. Infants with very large hemangiomas or miliary hemangiomatosis are also at risk for high- output cardiac compromise.

155
Q

156- Pachyonychia congenita type 2 is most commonly associated with which of the following:

A. Increased risk of malignancy
B. Poikiloderma
C. Natal teeth
D. Aplastic nails
E. Deafness

A

►C

Pachyonychia congenita type 2 may be associated with natal teeth and steatocystoma. Pachyonychia congenita type 1 is associated with benign leukoplakia.

156
Q

157- POEMS syndrome is associated with which of the following:

A. Premature aging
B. Odontogenic cysts
C. Eye abnormalities
D. M protein
E. Saddle nose deformity

A

►D

The acronym POEMS stands for polyneuropathy, organomegaly, endocrinopathy, M-protein, and skin changes.

157
Q

158 -A newborn presents with a well-defined, shiny patch with complete alopecia on the vertex of the scalp along the suture lines. Which of the following is the first step in diagnosis?

A. MRI
B. Calcium
C. Skin biopsy
D. Skull x-ray
E. Fungal culture

A

►A
The newborn most likely has aplasia cutis congenita. A MRI would be the best means of imaging to evaluate for any underlying bony abnormality. The infant does not need unnecessary radiation.

158
Q

9- A boy is noted at birth to have coarse scales over his trunk and extremities. The face, palms, soles and flexures are spared. What is the least likely association?

A. Corneal opacities
B. Cryptorchidism
C. Ectropion
D. Prolonged maternal labor
E. Neurologic abnormality

A

►C

X-linked ichthyosis is characterized by small, dark, firmly adherent scales accentuated on the sides of the neck and trunk. The face, palms, soles, antecubital and popliteal flexures are generally spared. Associated extracutaneous findings include corneal opacities (50%), undescended testes (20%), and prolonged maternal labor (usual). Neurological or mental retardation are rare but documented associations as XLI can be associated with a contiguous gene syndrome with Kallman syndrome, mental retardation and X linked recessive chrondrodysplasia punctata.

159
Q

160 -A 2 year-old has a high fever for three days, as the fever breaks, a generalized rash appears on the trunk. The most likely diagnosis is:

A. Measles
B. Mumps
C. Rubella
D. Erythema infectiosum
E. Roseola

A

►E

Roseola typically appears in toddlers. A high fever followed by a generalized maculopapular rash is characteristic for this HHV6 infection.

160
Q

161- What is the best test to confirm a diagnosis?

A. DsDNA antibody
B. Anti Ro antibody
C. Biopsy
D. KOH
E. Gram stain

A

►D

The picture depicts tinea faceii. A KOH exam should be performed to look for hyphae.

161
Q

162- This syndrome has severe aplasia cutis congenita, cutis marmorata telangiectatica congenita, limb defects and atrial septal defect:

A. Adams Oliver syndrome
B. Gianotti Crosti cynsrome
C. Castleman’s disease
D. Carrion’s disease
E. McCune Albright syndrome

A

►A

Adams Oliver syndrome is characterized by severe aplasia cutis congenita, cutis marmorata telangiectatica congenita, limb defects and atrial septal defect. Aplasia cutis congenital is characterized by an absence of skin and subcutaneous tissue.

162
Q

163- The lesion depicted is most often associated with:

A. Polyostotic fibrous dysplasia
B. Chondrodysplasia punctata
C. Sphenoid wing hypoplasia
D. Cleft palate
E. Osteopoikilosis

A

►A

The “coast of Maine” café au lait macule is associated with McCune-Albright syndrome and polyostotic fibrous dysplasia.

163
Q

164- Which of the following may be associated?

A. Paronychia
B. Cleft palate
C. AVM
D. Seizure disorder
E. Atrial septal defect

A

►D

Nevus sebaceus can very rarely be associated with multiple anomalies. Schimmelpenning syndrome can include seizure disorder, mental retardation, coloboma, as well as skeletal, cardiac and genitourinary abnormalities.

164
Q
  • Trichothiodystrophy classically includes which of the following findings:

A. Trichostasis
B. Trichoschisis
C. Pili torti
D. Trichorrhexis invaginata
E. Trichorrhexis nodosa

A

►B

Trichothiodystrophy or PIBIDS is associated with the “tiger-tail” abnormality of trichoschisis.

165
Q

166 -Which of the following is most likely associated?

A. No additional abnormality
B. Deafness
C. Coarctation of the aorta
D. Bifid rib
E. Alopecia

A

►A

The majority of patients with accessory tragii do not have any associated abnormalities. Multiple anomalies, including deafness and midline defects, have been reported but are rare.

166
Q

What is the most likely diagnosis?

A. Papular acrodermatitis of childhood
B. Mucocutaneous lymph node syndrome
C. German measles
D. Letterer-Siwe disease
E. Exanthem subitum

A

►A

Gianotti-Crosti syndrome is also known as papular acrodermatitis of childhood. The eruption is characterized by lichenoid papules in an acral distribution.

167
Q

168- A healthy, full-term infant develops a pustular, erythematous eruption on her face and trunk on the third day of life. A smear taken from one of these pustules would show:

A. Gram-positive bacteria
B. Predominantly neutrophils
C. Multi-nucleated giant cells
D. Predominantly eosinophils
E. Hyphae

A

►D

This baby has developed erythema toxicum neonatorum, a benign, self-limited eruption that occurs in the majority of healthy, full-term infants. Erythema toxicum usually develops on the second or third day of life and resolves by day 10. A smear taken from one of the pustules will demonstrate eosinophils, which is sufficient to make the diagnosis. Viral infections of the skin may demonstrate multi-nucleated giant cells on Tzanck smear. Transient neonatal pustular melanosis is present at birth; neonates present with small, superficial pustules that rupture easily. Some may have ruptured in utero, leaving pigmented macules. The pigmentation may persist for weeks to months in darkly- complexed infants. A smear of the intracorneal/subcorneal pustule will show mostly neutrophils, but eosinophils may also be present.

168
Q

169- What systemic abnormalities are associated with neonatal lupus?

A. Thrombocytopenia, transaminitis
B. Thrombocythemia, transaminitis
C. Anemia, hypocomplementemia
D. Thrombocytopenia, hypocomplementemia
E. Transaminitis, hypocomplementemia

A

►A

Thrombocytopenia and transaminitis are associated with neonatal lupus and may indicate a worse prognosis.

169
Q

170 -All 5 subtypes of Phakomatosis Pigmentovascularis have which feature in common:

A. Nevus spilus
B. Epidermal nevi
C. Nevus flammeus
D. Nevus anemicus
E. Dermal Melanocytosis.

A

►C

Phakomatosis Pigmentovascularis: Patients with a combination of vascular malformations and melanocytic or epidermal nevi are grouped into 4 subtypes of this disorder. All have nevus flammeus/capillary malformation (CM). Type I: CM + epidermal nevus Type II: CM + dermal melanocytosis +/- nevus anemicus Type III: CM + nevus spilus +/- nevus anemicus Type IV: CM + dermal melanocytosis + nevus spilus +/- nevus anemicus. (ie types II+III) Type V: CM + cutis marmorata

170
Q
  • Koplik spots typically appear:

A. Before the exanthem
B. At the same time as the exanthem
C. 1 week after the exanthem
D. 6 weeks after the exanthem
E. 8 weeks after the exanthem

A

►A

The enanthem of measles precedes the morbilliform eruption.

171
Q

172 -Which of the following should be the next step in the management of this patient?

A. Barium swallow
B. spine x-ray
C. Cardiac evaluation
D. Administration of oral antibiotics

E. Pulmonary function studies

A

►C

This patient must be evaluated thoroughly for PHACES syndrome. PHACES is an acronym for Posterior fossa malformations (Dandy-Walker malformation is most common), Hemangiomas, Arterial anomalies, Coarctation of the aorta, Eye abnormalities, and Sternal cleft defects. This patient should have a complete cardiac evaluation, neuroimaging, and ophthalmologic exam. If the facial hemangioma involves the beard area, this may indicate laryngeal involvement and appropriate imaging and evaluation is mandated. Propranolol is the treatment of choice however, caution must be taken if there is coarctation of the aorta and cardiology is often involved. Systemic steroids at high doses (5 mg/kg) can be considered if propranolol fails (it usually doesn’t). If caught early, the sequelae of PHACES syndrome can be minimized.

172
Q

173- A newborn presents with a pustular rash, joint swelling, oral mucosal lesions, and pain with movement. Over time, cutaneous pustulosis, ranging from discrete crops of pustules to generalized severe pustulosis and ichthyosiform lesions develop. In addition the newborn develops sterile multifocal osteomyelitis and periostitis. What is this condition?

A. Muckle-Wells syndrome
B. Deficiency of the interleukin-1-receptor antagonist (DIRA)
C. Familial Mediterranean fever
D. Hyper-IgD syndrome
E. PAPA syndrome

A

►B

This condition is deficiency of the interleukin-1-receptor antagonist (DIRA) and due to a mutation in the IL1RN gene. Muckle-Wells syndrome is due to a mutation in the CIAS1 gene and presents with sensorineural deafness, recurrent hives, and amyloidosis. Familial Mediterranean fever is most often due to a mutation in the MEFV gene. Hyper-IgD syndrome is a rare periodic fever syndrome characterised by attacks of fever, arthralgia, skin lesions including cyclical mouth ulcers, and diarrhea due to mutations in the mevalonate kinase gene. PAPA syndrome presents with pyogenic arthritis, pyoderma gangrenosum and acne and is associated with mutations in the CD2 binding protein 1 (CD2BP1).

173
Q

174- Hemangiomas of infancy are more likely to occur:

A. a. In males
B. b. In Asians
C. c. In children who are large for gestational age
D. d. In multiple gestation
E. e. In post term infants

A

►D

D. is Correct. Hemangiomas of infancy occur more commonly in girls, in Caucasians, in babies who are premature and low birth weight. They are more likely to occur in multiple gestation

174
Q

175- A patient presents with an epidermal nevus of the lower abdomen. You take a skin biopsy to confirm the diagnosis. The pathology report indicates that there were findings of epidermolytic hyperkeratosis. Which of the following defects is her offspring at risk for based on these findings?

A. Transglutaminase
B. Keratin 6a/16
C. Keratin 6b/17
D. Keratin 1/10
E. Keratin 2e
c

A

►D

Women with epidermal nevi with epidermolytic hyperkeratosis histologically are at risk for offspring with the “full-blown” epidermolytic hyperkeratosis syndrome, which is due to Keratin 1/10 defects. Transglutaminase defects are seen in lamellar ichthyosis, Keratin 6a/16 and 6b/17 in Pachyonychia congenita types I and II and Keratin 2e in Ichthyosis bullosa of Siemens.

175
Q

176- Posterior auricular adenopathy is a common feature of which exanthem:

A. Measles

B. Mumps
C. Scarlet fever
D. Rubella
E. Erythema infectiosum

A

►D

Lymphadenopathy with 1-7 days of malaise is commonly described with rubella infection. A rose- pink macular rash follows the prodrome.

176
Q

177- Which of followings syndromes characterized by Seborrheic-like or exfoliative dermatitis

A. Leiner’s disease
B. Hyperimmunoglobulinemia E syndrome
C. Wiskott-Aldrich syndrome
D. Chronic granulomatous disease
E. X-linked agammaglobulinemia

A

►A

Immunodeficiency disorders may be associated with a variety of cutaneous abnormalities, and recognition of these clinical features may allow an early diagnosis of primary immunodeficiency. Cutaneous abnormalities may include cutaneous infections, atopic- or seborrheic-like dermatitis, macular erythemas, alopecia, poor wound healing, purpura, petechiae, telangiectasias, pigmentary dilution, cutaneous granulomas, extensive warts, angioedema, and lupus-like changes. Leiner’s disease or erythroderma desquamativum is a complication of seborrheic dermatitis in infants (dermatitis seborrhoides infantum) There is usually a sudden confluence of lesions, leading to a universal scaling redness of the skin (erythroderma). The young patients are severely ill with anemia, diarrhea, and vomiting.The disease is both a familial and a nonfamilial form. The former is noted for having a functional deficiency of C5 complement, resulting in defective opsonization. These patients respond to antibiotics and infusions of fresh frozen plasma or whole blood. The other choices in the questions can all be associated with atopic -like dermatitis.

177
Q

178- A 24 month-old infant presents with yellowish-brown, crusted papules with petechiae in a seborrheic distribution. A biopsy is done to confirm a diagnosis. Which histologic picture is most likely?

A. CD1-, S100- cells with reniform nuclei
B. Foamy histiocytes with Touton giant cells
C. CD1+, S100+ cells with reniform nuclei
D. Mixed cellular infiltrate in a “ball and claw” pattern
E. Superficial perivascular infiltrate with mild spongiosis and neutrophil containing scale crust

A

►C

Langerhans cells are CD1 and S100 positive. The nuclei are described as kidney shaped, or reniform.

178
Q

179- A 4 month-old with diffuse blisters and erosions has a skin biopsy diagnostic of generalized mastocytosis. Which topical dressing should be avoided in this patient?

A. Mupirocin ointment
B. Polymyxin B ointment
C. Petrolatum
D. clobetasol ointment
E. Silver sulfadiazine

A

►B

Mastocytosis comprises a group of diseases characterized by increased number of mast cells in the skin and other organs. Seventy-five percent of cases occur before the age of 2. Patients with mastocytosis should avoid potential mast degranulators including aspirin, codeine, opiates, procaine, spicy foods, cheese, alcohol, polymyxin B.

179
Q

180- The disorder caused by a defect in intestinal zinc-specific transporter SLC39A4 is most likely to present:

A. At birth
B. Upon weaning from breast milk
C. Upon weaning from formula
D. In childhood
E. In adulthood

A

►B

In acrodermatitis enteropathica, there is decreased absorption of zinc from the infant”s gastrointestinal tract. The zinc in breast milk has greater availability than nonmaternal sources thus protecting the child from disease expression until weaning.

180
Q

181- What deficiency is responsible for this condition?

A. Thiamine
B. Niacin
C. Zinc
D. Vitamin C
E. Vitamin A

A

►C

Acrodermatitis enteropathica is a rare inherited disorder transmitted in an autosomal recessive fashion. The disorder is caused by an inability to absorb zinc. the clinical syndrome is characterized by acral dermatitis, alopecia, and diarrhea. Zinc deficiency from other causes (including chronic wasting, poor oral intake, lack of supplementation in total parenteral nutrition) can cause similar clinical changes.

181
Q

2 -The most likely etiology of Jacquet”s diaper dermatitis is:

A. Candida
B. Trichophyton rubrum
C. Group A beta-hemolytic streptococcus
D. Herpes simplex virus, Type 2
E. Multifactorial

A

►E

Jacquet”s diaper dermatitis is a multifactorial process. Yeast, irritants and moisture all contribute to the occurrence of this eruption.

182
Q

183- The Carney complex is associated with a defect in:

A. PRKAR1A
B. LYST
C. PTEN
D. MASH2
E. MLH1

A

►A

A PRKAR1A gene defect is associated with the Carney complex.

183
Q

184 -Which of the following is not true about Gianotti-Crosti?

A. It is associated with viral infections like enterocirus, EBV, and CMV
B. It typically affects children between the age of 3 months and 15 years
C. It is characterized by monomorphic symmetric flat topped papules in acral areas
D. It may have associated fevers, lymphadenopathy, and diarrhea
E. Corticosteroids should be given to alleviate the pruritus of the lesions

A

►E

Gianotti-Crosti, or papular acrodermatitis of childhood, is typically nonpruritic and corticosteroids should be avoided as they may have adverse effect. All the other statements are true. The condition typically resolves after 2-3 weeks.

184
Q

185- What disorder is associated with a defect in LMX1B?

A. Nail patella syndrome
B. Steatocystoma
C. Monilithrix
D. Lhermitte-Duclos syndrome
E. Chediak-Higashi syndrome

A

►A

A LMX1B gene defect is associated with nail patella syndrome.

185
Q

186 -A full term neonate is noted to have small pustules with no underlying erythema present at delivery. The pustules are easily removed with clearing of the vernix and a collarette appears. A gram stain is done showing predominately neutrophils without bacteria. What is the most likely diagnosis?

A. Miliaria
B. Erythema toxicum neonatorum
C. Transient neonatal pustular melanosis

D. Congenital candidiasis
E. Urticaria pigmentosa

A

►C

Transient neonatal pustular melanosis typically begins with sterile pustules that leave a characteristic collarette when ruptured. The lesions heal with hyperpigmented macules.

186
Q

187- Which finding is associated with Pachydermoperiostosis:

A. Cutis verticis gyrata
B. Palmoplantar hyperkeratosi
C. Mucosal keratoses
D. Osteopathia striata
E. Cutis marmorata

A

►A

The scalp change seen in pachydermoperiostosis is cutis verticis gyrata.

187
Q

188- What is the best treatment option?

A. Oral cephalexin
B. Oral acyclovir
C. Observation

D. Topical tretinoin
E. Topical ketoconazole

A

►D

The lesions shown in the photo are papules of infantile acne which occurs between the ages of 6mo to three years. The infants get comedones, papules and pustules. Topical tretinoin would be the best treatment option.

188
Q

189- This patient had significantly elevated serum CPK. The likely diagnosis is:

A. Dermatomyositis
B. Lupus erythematosus
C. Psoriasis
D. Atopic dermatitis
E. Lichen planus

A

►A

The answer is dermatomyositis with the heliotrope color and distribution of erythema or violaceous color. The skin over the metacarpal and proximal interphalangeal joints can become inflamed and erythematous forming Gottron‟s papules.

189
Q

190- Most common malignancy associated with multiple lesions similar to the attached image is:

A. Acute myelogenous leukemia
B. Chronic myelogenous leukemia
C. Acute lymphocytic leukemia
D. Chronic lymphocytic leukemia
E. Melanoma

A

►B

The image shown is juvenile xanthogranuloma. The most commonly associated malignancy is chronic myelogenous leukemia. Other associations include Neurofibromatosis type 1 and aquagenic pruritus.

190
Q

191 -A neonate presents with a large segmental hemangioma of the V1 distribution. What are features may be associated with this finding?

A. Tram-track calcifications
B. Anterior fossa defect
C. Posterior fossa defect
D. M-paraproteinemia
E. Muscular dystrophy

A

►C

PHACES syndrome consists of posterior fossa defect, hemangiomas (often segmental and large), arterial defects, cardiac defects, eye abnormaliteis, and sternal clefting. Posterior fossa defects include the Dandy-Walker malformtion.

191
Q
    • Schimmelpenning-Feuerstein-Mims syndrome may be associated with which of the following:

A. Osteopokilosis
B. Polyostotic fibrous dysplasia
C. Osteopathia striata
D. Chondrodysplasia punctata
E. Hypophosphatemic rickets

A

►E

Linear nevus sebaceous syndrome, also known as Schimmelpenning-Feuerstein-Mims syndrome, may be associated with hypophospatemic, vitamin D resistant rickets.

192
Q

193 -GLUT-1 stains placental tissue. What other tissue shows positive GLUT-1 staining?

A. capillary vascular malformation
B. salmon patch
C. glomus tumor
D. infantile hemangioma
E. angel’s kiss

A

►D

The correct answer is infantile hemangioma, which reliably displays GLUT-1 positivity. GLUT1 stains infantile hemangioma with high sensitivity and specificity. Glomus tumor, capillary vascular malformation (also called port-wine stain), salmon patch, and angel’s kiss are other vascular lesions that do not stain with GLUT-1.

193
Q

194- Hemangiomas of infancy are more likely to be:

A. GLUT-1 negative
B. Merosin positive but GLUT-1 negative
C. Lewis Y antigen positive, merosin negative
D. Glut-1 positive, merosin positive, Lewis Y antigen negative
E. Glut-1 positive, merosin positive, Lewis Y antigen positive

A

►E

Hemangiomas show GLUT-1 positivity and stain positively with placental markers.

194
Q

195- A 3 month-old girl with multiple hemangiomas along her right jaw is at increased risk for:

A. Bleeding complications
B. Underlying bone abnormalities
C. Subglottic hemangioma
D. Oral obstruction
E. Hearing defecits

A

►C

Hemangiomas are benign vascular tumors which have proliferating phase and then a spontaneous involution phase. Depending upon the location, some hemangiomas may have more long term sequelae such as scarring or structural malformation, and rarely, consumptive coagulopathy. Infants with hemangiomas of the “beard” distribution should be evaluated for subglottic hemangiomas which may lead to airway obstruction.

195
Q

196- A deficiency of the surface glycoprotein sialophorin is seen in which immunedeficient disease?

A. Wiskott-Aldrich syndrome
B. Chronic granulomatous disease
C. Job syndrome

D. Severe combined immunodeficiency syndrome
E. Leiner”s disease

A

►A

Wiskott-Aldrich syndrome is an X-linked recessive disorder caused by mutations in WAS gene. Patients with Wiskott-Aldrich may have unstable sialoglycoprotein CD 43 on the surface of lymphocytes. Patients with Wiskott-Aldrich have atopic dermatitis with increased risks for secondary infection, thrombocytopenia, and recurrent bacterial infections. They are also at an increased risk for lymphoreticular malignancy (20%).

196
Q

197- Which treatment choice would be contraindicated in a one-year old child who presents with monomorphous, nonpruritic flat-topped papules on the face, buttocks, extremities, palms and soles?

A. Advil
B. Acetaminophen
C. Hydration
D. Corticosteroids
E. Observation

A

►D

Gianotti-Crosti or papular acrodermatitis of childhood is associated with a variety of viral infections. Patients have a typical cutaneous manifestation, low-grade fever, mild lymphadenopathy and diarrhea. Corticosteroids should be avoided as they may have an adverse effect.

197
Q

198 -An eight year-old boy presents with pink, flushed cheeks and a low-grade fever. Eruptions then appeared. What is the most likely etiology?

A. Paramyxovirus
B. Togavirus
C. Group A streptococcus
D. HHV6

E. Parvovirus

A

►E

Fifths Disease or erythema infectiosum, is caused by parvovirus B19, a single-stranded DNA virus. There are a few clinical presentations including, diffuse lacy rash on the trunk that spreads gradually toward the distal extremities, papular-pruritic “gloves-and-socks” syndrome and
aplastic crisis which does not have concomitant rash.

198
Q

199 -A patient presents with multiple juvenile xanthogranulomas, axillary freckling, multiple café- au-lait macules, three neurofibromas and a family history of NF-1. What other condition is this patient at increased risk for?

A. AML
B. JMML
C. CLL
D. Medulloblastoma
E. Pancreatic carcinoma

A

►B

A patient with Neurofibromatosis type I and JXG‟s present is at increased risk for JMML. Medulloblastomas are increased in patients with Basal Cell Nevus syndrome. Pancreatic carcinoma occurs at an increased risk in patients with mutations in CDKN2A in the familial melanoma syndrome.

199
Q

200 -What is the most likely diagnosis:

A. Epidermolysis bullosa simplex
B. Epidermolysis bullosa dystrophica
C. Pemphigus vulgaris
D. Linear bullous IgA disease
E. Facticial dermatosis

A

►A

The bullous lesions represented are nonscarring and are associated with obvious nail dystrophy. Epidermolysis bullosa simplex, Dowling-Meara type, is the best option.

200
Q
A